You are on page 1of 87

UPSC ANALYSIS DOCUMENT FOR

PRELIMS
GEOGRAPHY ANALYSIS DOCUMENT (2011-22)
HOW TO READ NCERT (6TH TO 12TH IN CONSOLIDATION)
(PART OF PRELIMS MARATHON 2023 and
the Mentorship Batch 2022-23)
BY
ER NEELESH KUMAR SINGH
ALL INDIA RANK 442 UPSC CSE 2021
JOIN ME AT TELEGRAM - UPSC PRELIMS WITH NEELESH
https://t. me/UPSCPrelimsWith Neelesh

JOIN ME AT TELEGRAM-UPSC PRELIMS WITH NEELESH, FOR FREE CSAT VIDEOS AND OTHERS-JOIN ME IN VOUTUBE CHANNEL-CIVIL SERVICES WITH
NEELESH. For Queries-Whatsapp-9310161970, Website-neeleshair442.com, www.csetopper.com for best test series (Our other initiative-Free
Sociology, Free Ethics and Essay, Free CSAT) ©Copyright NEELESH KUMAR SINGH (AIR 442 UPSC CSE 2021)
UPSC ANALYSIS DOCUMENT FOR PRELIMS
GEOGRAPHY ANALYSIS
ON
HOW TO READ NCERT CLASS 6 TO 12 IN CONSOLIDATION FOR UPSC THROUGH PYQ (2011-22)
(PART OF OUR PRELIMS MARATHON)
BY

ER NEELESH KUMAR SINGH


ALL INDIA RANK 442 UPSC CSE 2021
(UNDERSTANDING UPSC DEMAND THROUGH PYQ 2011-2022)
JOIN ME AT TELEGRAM – UPSC PRELIMS WITH NEELESH

JOIN ME AT TELEGRAM – UPSC PRELIMS WITH NEELESH (AIR 442 UPSC CSE 2021)
https://t.me/UPSCPrelimsWithNeelesh
VISIT OUR WEBSITE – neeleshair442.com ©Copyright reserved with author
INDEX FOR GEOGRAPHY DOCUMENT
TOPIC NUMBER TOPIC NAME NUMBER OF
QUESTIONS
TOPIC 1 INDIA: LOCATION, STRUCTURE AND PHYSIOGRAPHY 19
TOPIC 2 INDIA : DRAINAGE SYSTEM 12
TOPIC 3 INDIA: CLIMATE 4
TOPIC 4 INDIA: NATURAL VEGETATION 12
TOPIC 5 INDIA: SOILS 3
TOPIC 6 INDIA: NATURAL HAZARDS AND DISASTER 0
TOPIC 7 INDIA: POPULATION, MIGRATION, HUMAN DEVELOPMENT, HUMAN SETTLEMENT 0
TOPIC 8 INDIA: LAND RESOURCES AND AGRICULTURE 12
TOPIC 9 INDIA: WATER RESOURCES 4
TOPIC 10 INDIA: MINERAL AND ENERGY RESOURCES 7
TOPIC 11 INDIA: MANUFACTURING INDUSTRIES 1
TOPIC 12 INDIA: SERVICE SECTOR (TRANSPORT AND COMMUNICATION AND INTERNATIONAL TRADE ETC) 1
TOPIC 13 INDIA: PLANNING AND SUSTAINABLE DEVELOPMENT IN INDIAN CONTEXT AND GEOGRAPHICAL PERSPECTIVES 0

TOPIC 14 INDIA MISCELLANEOUS (NOT FROM NCERT THOUGH) 3


TOPIC 15 WORLD: GEOGRAPHY AS A DISCIPLINE 0
TOPIC 16 WORLD: THE EARTH 6
TOPIC 17 WORLD: LANDFORMS 1
TOPIC 18 WORLD: CLIMATE 13
TOPIC 19 WORLD: WATER (OCEAN) 6
TOPIC 20 WORLD: LIFE ON THE EARTH 5
TOPIC 21 WORLD: HUMAN RESOURCES 0
TOPIC 22 WORLD: RESOURCES 0
TOPIC 23 WORLD: PRIMARY ACTIVITIES 5
TOPIC 24 WORLD SECONDARY ACTIVITIES 0
TOPIC 25 WORLD TERTIARY AND OTHER SECTORS 0
TOPIC 26 Map QUESTIONS (LOCATION BASED) (NOT PROPERLY GIVEN IN NCERT) 16

JOIN ME AT TELEGRAM – UPSC PRELIMS WITH NEELESH (AIR 442 UPSC CSE 2021)
https://t.me/UPSCPrelimsWithNeelesh
VISIT OUR WEBSITE – neeleshair442.com ©Copyright reserved with author
GEOGRAPHY ANYLYSIS (BOTH INDIAN AND WORLD BASED ON NCERT)
In this analysis, I have tried to show how to read NCERT. Normally most of the time we hear people/teachers/mentors etc. say – Read NCERT
which is a very good advice, but at the same time, you should know what kind of questions are framed from the NCERT Geography. It helps you
in a directed preparation. Further, since it is advisable to read NCERT of Geography from Class VI to Class XII, it is better, if we read them in
integrated form rather than simply reading class wise. This will ensure, that you cover same topics from difference classes and may cut /remove
the overlapping so that, it can assist you during revision. Remember, there is no substitute for the consolidated preparation. We will move in
our PRELIMS BATCH based on this only. So, we will be reading chapters from different classes simultaneously for consolidated preparation. So,
let us arm ourselves with the knowledge of NCERTs.
Please note – Many chapters are overlapping. So, I have tried to club them at the places where I think, it will be best for it. However, if there is
any scope of better categorization, your advices are welcome.
Simply follow the step-by-step procedure
1. Before reading any chapter, first read the Previous Year Questions asked from the chapter. At this stage, do not see the solution for
maximum benefit
2. Read the Instructions related to the chapter before reading the actual chapter which has been prepared by me based on the analysis
of 2011-2022 UPSC Question. Remember you must not ignore the instructions related to the chapters.
3. Now read the actual chapter from the book (NCERT CLASS 6TH TO 12TH Selective chapters for each segment) keeping in mind the PYQ
questions and my instructions. For a better direction, all the chapters of NCERT Geography have been categorized from Class VI to Class XII.
4. Come back to the original questions again and then solve the questions. Prepare the peripheral areas if any. Infact, whenever you read
your current affairs related to geography, keep these questions in mind. It will assist in directed preparation.
5. See the solution and see how many you solved correctly.
6. Work on your weak areas if any in the chapter as per your attempt
7. Congratulations, now you are the master of this chapter
8. It is advised that you must not ignore the MAPS given in your NCERTs. Do not try to prepare maps in isolation. It is always better that
you should complete the related MAP portion when you are reading the topic. Though, you need to supplement maps by referring your ATLAS.
Please note – When I say, some part is important, it does not mean that you should completely ignore other things in the chapter. It is just
that you must not leave those specific topics/concepts which are mentioned in the instruction explicitly.

JOIN ME AT TELEGRAM – UPSC PRELIMS WITH NEELESH (AIR 442 UPSC CSE 2021)
https://t.me/UPSCPrelimsWithNeelesh
VISIT OUR WEBSITE – neeleshair442.com ©Copyright reserved with author
CIVIL SERVICES WITH NEELESH
(TELEGRAM CHANNEL – UPSC PRELIMS WITH NEELESH)
(YOUTUBE CHANNEL – CIVIL SERVICES WITH NEELESH)
Note – All our initiatives (both free and paid) are on these channels only.

OUR INITIATIVES (Running Currently)

1. PRELIMS MENTORSHIP 2024 (BOTH FREE AND PAID)


2. ETHICS AND ESSAY EVALUATION – ER NEELESH AIR 442 UPSC_CSE2021 (https://t.me/UpscWithNeelesh_AIR442)
3. SOCIOLOGY FRAMEWORK - TELEGRAM – SOCIOLOGY WITH NEELESH (https://t.me/SociologyWithNeelesh)
4. FREE CSAT PROGRAM – ON YOUTUBE CHANNEL – CIVIL SERVICES WITH NEELESH
5. INTEGRATED MENTORSHIP FOR 2025 (PRELIMS AND MAINS) – YEARLONG MENTORSHIP
6. SHORT NOTES FOR ALL SUBJECTS (HANDWRITTEN)

UPCOMING INITIATIVES

1.PRELIMS TEST SERIES FOR 2025


2. MAINS TEST SERIES FOR 2025
3. ESSAY VIDEO BATCH FOR 2024 AND 2025
4. ETHICS VIDEO BATCH FOR 2024 AND 2025
5. SOCIOLOGY VIDEO BATCH FOR 2025 BASED ON PYQ
6. CSAT VIDEO BATCH
7. MAINS MARATHON 2024 AND MUCH MORE
INDIA
TOPIC NAME - INDIA: LOCATION, STRUCTURE AND PHYSIOGRAPHY
CLASS XI: Physical Environment Chapter 1 (Location) + Chapter 2
(Structure and Physiography)
CLASS IX Chapter 1 (India – Size and Location)
+ Chapter 2 (Physical Features of
India)
CLASS VI Chapter 7 (Our Country – India)

Total Number of Questions Asked from 2011-22 19


My Instructions related to these Chapters Look for the PYQ. This portion of chapters is one of the favorite areas
of UPSC. So, they tend to make the questions on any line. Also, look
for the spread, location of various physiographic divisions/portions
such as hills, mountains, coastal plains, western ghats etc. Know their
orientation and relative location etc. In short, you may say,
everything in these chapters is important for UPSC.
You are advised to look for ATLAS too while reading these chapters
beside looking for map given in NCERT. This will ease your anxiety fo
Maps.

PREVIOUS YEAR QUESTIONS RELATED TO THESE CHAPTERS OR TOPICS


S.No. Question Answer Explanation
1 Which one of the following pairs of States of India d Easternmost state of India is Arunachal Pradesh and
indicates the easternmost and westernmost State? (2015) Westernmost state of India is Gujarat.
a. Assam and Rajasthan

JOIN ME AT TELEGRAM – UPSC PRELIMS WITH NEELESH (AIR 442 UPSC CSE 2021)
https://t.me/UPSCPrelimsWithNeelesh
VISIT OUR WEBSITE – neeleshair442.com ©Copyright reserved with author
b. Arunachal Pradesh and Rajasthan
c. Assam and Gujarat
d. Arunachal Pradesh and Gujarat

2 When you travel in Himalayas, you will see the following: d Himalayan mountains are the young fold mountains, that
(2012) has all the geographic features mentioned above.
1. Deep gorges.
2. U-turn river courses. Just go through the said features in detail
3. Parallel mountain ranges.
4. Steep gradients causing land-sliding
Which of the above can be said to be the evidences for
Himalayas being young fold mountains?
a. 1 and 2 only
b. 1,2 and 4 only
c. 3 and 4 only
d. 1,2,3 and 4

3 A state in India has the following characteristics: (2011) b The north part of Gujarat shares its climate with Rajasthan
1. Its northern part is arid and semi-arid. and can be termed as semi-arid and arid type of a climate.
2. Its central part produces cotton. The south of the state gets its fair share of the oncoming
3. Cultivation of cash crops is predominant over food monsoons.
crops. Gujarat is premier in cotton producing states in the country.
Which one of the following states has all of the above Moreover, cash crops are predominantly produced in
Gujarat.
characteristics?
a. Andhra Pradesh
b. Gujarat
c. Karnataka
d. Tamil Nadu

JOIN ME AT TELEGRAM – UPSC PRELIMS WITH NEELESH (AIR 442 UPSC CSE 2021)
https://t.me/UPSCPrelimsWithNeelesh
VISIT OUR WEBSITE – neeleshair442.com ©Copyright reserved with author
4 Consider the following statements: (2017) b The Indian Himalayan Region (IHR) is the section of the
1. In India, the Himalayas are spread over five States Himalayas within India, spanning the states of Jammu &
only. Kashmir (UT), Himachal Pradesh, Uttarakhand, Sikkim,
2. Western Ghats are spread over five states only. Arunachal Pradesh, as well as the hilly regions of two states
3. Pulicat lake is spread over two States only. – Assam and West Bengal. Hence, statement 1 is not correct.
Which of the statements given above is/are correct?
a. 1 and 2 only Western Ghats is a mountain range that runs through the
b. 3 only states of Gujarat, Maharashtra, Goa, Karnataka, Kerala and
c. 2 and 3 only Tamil Nadu (total 6 states). Hence, statement 2 is also not
d. 1 and 3 only correct.

Pulicat Lake formerly Pralaya Kaveri is the second largest


brackish water lake or lagoon in India, after Chilka Lake. It is
at the border of two states Andhra Pradesh and Tamil Nadu,
being situated on the Coromandel Coast in South India.
Hence, only statement 3 is correct.

5 Siachen Glacier is situated to the: (2020) d Siachen glacier is the glacier located in the eastern
a. East of Aksai Chin Karakoram range in the Himalayas, just northeast of the
b. East of Leh point NJ9842 where the Line of Control between India and
c. North of Gilgit Pakistan ends. It is located at the North of Nubra valley
d. North of Nubra Valley which acts as the gateway to the Siachen Glacier and
Karakoram Pass.
Siachen glacier is situated to the west of Aksai Chin, north of
Leh and to the east of Gilgit. So (a), (b), and (c) are not
correct.

6 A particular State in India has the following characteristics: a Arunachal Pradesh and Northern Rajasthan are located at
(2012) the same latitude i.e., 26°.
1. It is located on the same latitude which passes As per ISFR 2021, 79.33 percent area of Arunachal Pradesh
through northern Rajasthan. is under the forest cover.

JOIN ME AT TELEGRAM – UPSC PRELIMS WITH NEELESH (AIR 442 UPSC CSE 2021)
https://t.me/UPSCPrelimsWithNeelesh
VISIT OUR WEBSITE – neeleshair442.com ©Copyright reserved with author
2. It has over 80% of its area under forest cover. Over 12% of forest cover constitutes Protected Area
3. Over 12% of forest cover constitutes Protected Network in this state (This data is of 1999 available on
Area Network in this State. https://arunachalforests.nic.in/).
Which one among the following States has all the above
characteristics? These kinds of questions can be seen often. To get it correct,
a. Arunachal Pradesh just develop your understanding on the detailing involved.
b. Assam Like if you are studying about any state, mark out the
c. Himachal Pradesh important feature of that state.
d. Uttarakhand
7 From the ecological point of view, which one of the a Sathyamangalam Wildlife Sanctuary and Tiger Reserve is a
following assumes importance in being a good link protected area and tiger reserve along the Western Ghats in
between the Eastern Ghats and the Western Ghats? the Indian State of Tamil Nadu. Sathyamangalam forest
(2017) range is a significant wildlife corridor in the Nilgiri Biosphere
a. Sathyamangalam Tiger Reserve Reserve between the Western Ghats and the rest of the
b. Nallamalla Forest Eastern Ghats and a genetic link between the four other
c. Nagarhole National Park protected areas which it adjoins, including the Billigiriranga
d. Seshachalam Biosphere Reserve Swamy Temple Wildlife Sanctuary, Sigur Plateau, Mudumalai
National Park and Bandipur National Park

8 Which one of the following pairs of islands is separated a Ten Degree channel separates the South Andaman and Car
from each other by the ‘Ten Degree Channel’? (2014) Nicobar in the Bay of Bengal.
a. Andaman and Nicobar
b. Nicobar and Sumatra
c. Maldives and Lakshadweep
d. Sumatra and Java

9 At one of the places in India, if you stand on the seashore c The answer is Chandipur.
and watch the sea, you will find that the sea water recedes These are the exceptional questions Simply know and move
from the shore line a few kilometers and comes back to forward.
the shore, twice a day, you can actually walk on the sea

JOIN ME AT TELEGRAM – UPSC PRELIMS WITH NEELESH (AIR 442 UPSC CSE 2021)
https://t.me/UPSCPrelimsWithNeelesh
VISIT OUR WEBSITE – neeleshair442.com ©Copyright reserved with author
floor when the water recedes. This unique phenomenon
is seen at: (2017)
a. Bhavnagar
b. Bheemunipatnam
c. Chandipur
d. Nagapattinam

10 Consider the following statements: (2018) a Barren island is located in the Andaman Sea, only active
1. The Barren Island volcano is an active volcano volcano in South Asia and the only active volcano along a
located in the Indian territory. chain of volcanoes from Sumatra to Myanmar. Along with
2. Barren island lies about 140 km east of Great the rest of the Andaman Islands, it is a part of the Indian UT
Nicobar. of Andaman and Nicobar Islands and is located in the east of
3. The last time the Barren Island volcano erupted Middle Andaman. Hence, 1 is correct and 2 is incorrect.
was in 1991 and it has remained inactive since then. The first recorded eruption of the volcano dates back to
Which of the statements given above is/are correct? 1787. Since then, the volcano has erupted more than ten
(a) 1 only times, with the most recent one being in January 2017.
(b) 2 and 3 Hence, 3 is incorrect.
(c) 3 only
(d) 1 and 3

11 Consider the following pairs: (2022) b - Namcha Barwa is a mountain peak lying in Tibet in the
Peak Mountains region of Pemako. It is not in the Garhwal Himalayas (which
1. Namcha Barwa : Garhwal Himalayas are located in Uttarakhand).
2. Nanda Devi : Kumaon Himalaya - Nanda Devi is second-highest mountain in India after
3. Nokrek : Sikkim Himalaya Kangchenjunga, and the highest peak located entirely within
Which of the pairs given above is/are correctly matched? the country. Nanda Devi peak forms a part of the Kumaon
a. 1 and 2 Himalayas. The Kumaon Mountains are the part of the
b. 2 only northern Himalayas which is situated 320 km away from the
c. 1 and 3 Sutlej River and is in the east of Kali River. So, pair 2 is
d. 3 only correctly matched.

JOIN ME AT TELEGRAM – UPSC PRELIMS WITH NEELESH (AIR 442 UPSC CSE 2021)
https://t.me/UPSCPrelimsWithNeelesh
VISIT OUR WEBSITE – neeleshair442.com ©Copyright reserved with author
- Nokrek is the highest peak in the Garo Hills range with an
elevation of 1412 metres above sea level, from where rise
most of the major rivers and streams in the Garo Hills region.
Hence, (b) is the correct answer.

12 Consider the following pairs: (2014) c Cardamom Hills are found in the parts of Western Ghats
Hills Region located in the states of Kerala and Tamil Nadu. Whereas the
1. Cardamom Hills : Coromandel Coast coromandel coast is shared by the states of Tamil Nadu,
2. Kaimur Hills : Konkan Coast Andhra Pradesh and the UT of Puducherry.
3. Mahadeo Hills : Central India Kaimur range is found the state of Madhya Pradesh, also
4. Mikir Hills : North-East India known as Kaimur Hills.
Which of the above pairs are correctly matched? Mahadeo hills is a mountain in the State of Madhya Pradesh
a. 1 and 2 (Central India).
b. 2 and 3 Mikir Hills are the hills in the State of Assam (North-East
c. 3 and 4 India).
d. 2 and 4

13 Consider the following pairs: (2015) a Srisailam is an ancient pilgrimage center situated on the
Place of pilgrimage Location Nallamalai Hills of Kurnool District of Andhra Pradesh. It is a
1. Srisailam : Nallamala Hills temple town situated on the southern bank of the river
2. Omkareshwar : Satmala Hills Krishna. It is famous for the Mallikarjun Swamy Jyotirlinga
3. Pushkar : Mahadeo Hills and many of its related temples dating back to the medieval
Which of the above pairs is/are correctly matched? times.
a. 1 only Omkareshwar is one of foremost of the 12 Jyotirlinga or it is
b. 2 and 3 only luminous physical self-presentation of Lord Shiva of the
c. 1 and 3 only Hindus. It is located in Madhya Pradesh, on the Mandhata
d. 1,2 and 3 hill on the banks of the Narmada. On the other hand,
Satmala hills run across Nashik District of Maharashtra.
The pilgrimage place, Pushkar has the only temple for Lord
Brahma in India and the world, (Lord Brama is known as the
creator of the world as per the Hindu mythology). Pushkar

JOIN ME AT TELEGRAM – UPSC PRELIMS WITH NEELESH (AIR 442 UPSC CSE 2021)
https://t.me/UPSCPrelimsWithNeelesh
VISIT OUR WEBSITE – neeleshair442.com ©Copyright reserved with author
lies on the shore of the sacred Pushkar Lake. Pushkar is in
Rajasthan while Mahadeo Hills are in Madhya Pradesh.

14 Consider the following pairs: (2016) c - Bagelkhand or Baghelkhand is a region and also a
Famous place Region mountain range in Central India that covers the
1. Bodhgaya : Baghel Khand northeastern regions of Madhya Pradesh and a small area
2. Khajuraho : Bundelkhand of western Uttar Pradesh. Bodhgaya is in Bihar.
3. Shirdi : Vidarbha - The Khajuraho group of monuments was built during the
4. Nasik (Nashik) : Malwa rule of the Rajput Chandela dynasty. The building activity
5. Tirupati : Rayalseema started almost immediately after the rise of their power,
Which of the pairs given above are correctly matched? throughout their kingdom to be later known as
a. 1, 2 and 4 Bundelkhand.
b. 2,3.4 and 5 -Vidarbha is the eastern region of the Indian state of
c. 2 and 5 only Maharashtra, comprising Nagpur Division and Amravati
d. 1,3,4 and 5 Division. Shirdi is a town and falls under Ahmednagar District
in Maharashtra. It is not in Vidarbha region.
- Nasik is not in the Malwa region.
- Tirupati lies in the Rayalaseema region of Andhra Pradesh.

15 If you travel by road from Kohima to Kottayam, what is the b In order to travel by road from Kohima to Kottayam, we
minimum number of States within India through which need to cross Nagaland (origin), Assam, West Bengal,
you can travel, including the origin and destination? Odisha, Andhra Pradesh and then either through Tamil Nadu
(2017) to Kerala (Kottayam – destination) or through Andhra
a. 6 Pradesh to Karnataka to Kerala; So, we need to cross Seven
b. 7 states either way.
c. 8
d. 9

16 Which of the following is geographically closest to Great a Great Nicobar is the southernmost and largest of the
Nicobar? (2017) Nicobar Islands of India north of island Sumatra which is
a. Sumatra located about 1192 km to south of Great Nicobar.

JOIN ME AT TELEGRAM – UPSC PRELIMS WITH NEELESH (AIR 442 UPSC CSE 2021)
https://t.me/UPSCPrelimsWithNeelesh
VISIT OUR WEBSITE – neeleshair442.com ©Copyright reserved with author
b. Borneo Distance between Great Nicobar to Borneo: 2398 km.
c. Java Distance between Great Nicobar to Java: 2484 km
d. Sri Lanka Distance between Great Nicobar to Sri Lanka: 1437

17 Among the following cities, which one lies on a longitude a Both Delhi and Bengaluru fall on the same longitude.
closest to that of Delhi? (2018) State Longitude
a. Bengaluru Delhi 77.21 degree E
b. Hyderabad Bengaluru 77.56 degree E
c. Nagpur Hyderabad 78.48 degree E
d. Pune Nagpur 79.08 degree E
Pune 73.84 degree E

18 Which one of the following is an artificial lake? (2018) a Kodaikanal lake is a manmade lake located in the Kodaikanal
a. Kodaikanal (Tamil Nadu) city in Dindigul district in Tamil Nadu. Sir Vere Henry Levinge,
b. Kolleru (Andhra Pradesh) the then Collector of Madurai, was instrumental in creating
c. Nainital (Uttarakhand) the lake in 1863, amidst the Kodaikanal town which was
d. Renuka (Himachal Pradesh) developed by the British and early missionaries from USA.
The lake is star-shaped, centrally located in the town of
Kodaikanal and is surrounded by lush green hills of the
north-western Palani hill range, which is the main watershed
for the lake.

Kolleru lake forms the largest shallow fresh water lake in


Asia. It is located in Andhra Pradesh.

Nainital lake, a natural freshwater body, situated amidst the


township of Nainital in Uttarakhand, tectonic in origin, is
kidney shaped or crescent shaped and has an outfall at the
southeastern end.

JOIN ME AT TELEGRAM – UPSC PRELIMS WITH NEELESH (AIR 442 UPSC CSE 2021)
https://t.me/UPSCPrelimsWithNeelesh
VISIT OUR WEBSITE – neeleshair442.com ©Copyright reserved with author
Renuka lake at Sirmaur, Himachal Pradesh, is the largest
natural lake in the state. The lake is named after the goddess
Renuka.

19 With reference to India, Didwana, Kuchaman, Sargol and d There are many saline – water lakes in Rajasthan out of
Khatu are the names of: (2021) which Sambhar (Jaipur). Didwana (Nagaur), Pachpadra
a. Glaciers (Barmer), Lunkaransar (Bikaner), Kuchaman (Nagaur),
b. Mangrove Areas Phalodi (Jodhpur), Kavod (Jaisalmer) are famous.
c. Ramsar Sites
d. Saline Lakes Kuchaman lake is a continental saline lake located around
Kuchaman City, Nagaur district, Rajasthan. It is about 8.5 sq.
Kms in area.

Small lakes with flat floors and undrained basins in which


water collects after rains and evaporates quickly are called
playas. The Didwana is a playa. Other such playas are the
Kuchaman, the Sargol and the Khatu lakes.

Additional Information
1. India is the seventh largest country – Russia>Canada>USA>China>Brazil>Australia>India> Argentina
2. Northernmost point of India (Indira Col – located in Ladakh), Southernmost point of India (Indira Point of Great Nicobar Island of
Andaman and Nicobar Islands. In case of India’s mainland, it is Kanyakumari of Tamil Nadu), Easternmost point of India (Kibithu in
Arunachal Pradesh) and Westermost point of India (Guhar Moti in Gujarat)
3. Bay of Bengal, Arabian Sea and India Ocean meet at Kanyakumari.
4. As per Census 2011, the world’s 17.5 percent population lives in India.
5. Mahi river is the only river in India that cuts Tropic of Cancer twice (First in MP and then Rajasthan)
6. Udaipur City of Tripura is closest to the Tropic of Cancer.
7. Tropic of Cancer passes through Gujarat, Rajasthan, MP, Chhattisgarh, Jharkhand, West Bengal, Tripura and Mizoram (total 8 states)
8. Indian Standard time (82o30’ E) passes through Naini Area of Prayagraj in the state of Uttar Pradesh. It overall passes through 5 Indian
states which are Uttar Pradesh, MP, Chhattisgarh, Odisha and Andhra Pradesh.

JOIN ME AT TELEGRAM – UPSC PRELIMS WITH NEELESH (AIR 442 UPSC CSE 2021)
https://t.me/UPSCPrelimsWithNeelesh
VISIT OUR WEBSITE – neeleshair442.com ©Copyright reserved with author
9. The Barren island, the only active volcano is situated in the east of the Middle Andaman,
10. 10 degree channel separates the Andaman from Nicobar.
11. Duncan Pass is situated between South Andaman and Little Andaman.
12. Coco Strait is between Coco Islands (Myanmar) and North Andaman.
13. 6 degree channel or the Great Channel is located between India (Indira Point) and Indonesia (Rondo Island) near 6 degree latitude. To
the south of Rondo Island is the island of Sumatra, Indonesia.
14. 8 degree channel is between Minicoy (Lakshadweep) and Maldives.
15. 9 degree channel separates Minicoy from the main archipelago of Lakshadweep.
16. There are landlocked 5 states and 2 UT which do not touch international border (Haryana, Madhya Pradesh, Chhattisgarh, Jharkhand,
Telangana, Delhi (UT) and Chandigarh (UT).
17. Puducherry is spread in 3 states (Puducherry + Karaikal in TAMIL NADU, Yanam in Andhra Pradesh and Mahe in Kerela).
18. Frontiers of India and Indian states touching them Bangladesh. (in decreasing order of length)

Neighboring Country Length of common Number of Indian States/UT Name of Those States/UT
border having common border
Bangladesh 4096.7 5 WB, Assam, Meghalaya, Tripura, Mizoram
China 3488 5 Ladakh (UT), Himachal Pradesh,
(Note – Jammu and Kashmir now Uttarakhand, Sikkim, Arunachal Pradesh
does not touch China)
Pakistan 3323 5 Jammu and Kashmir (UT), Ladakh (UT),
Punjab, Rajasthan, Gujarat
Nepal 1751 5 Uttarakhand, UP, Bihar, WB, Sikkim
Myanmar 1643 4 Arunachal Pradesh, Nagaland, Manipur,
Mizoram
Bhutan 699 4 Sikkim, WB, Assam, Arunachal Pradesh
Afghanistan 106 1 Ladakh (UT) (POK)

19. India and Pakistan (Radcliff Line), Durand Line (India and Afghanistan), Mcmohan Line( India and China), Maginot Line (France and
Germany)
20. Kuttanad or Kuttanadu of Kerela has the lowest altitude in India. It is also known as the Rice Bowl of India
21. The famous Aksai Chin is in Ladakh Region

JOIN ME AT TELEGRAM – UPSC PRELIMS WITH NEELESH (AIR 442 UPSC CSE 2021)
https://t.me/UPSCPrelimsWithNeelesh
VISIT OUR WEBSITE – neeleshair442.com ©Copyright reserved with author
22. The oldest mountain range in India is Aravalli (these are residual mountains)
23. Highest Peak of Southern India is Anaimudi and is a part of Western Ghats
24. K2 is the highest peak in India (also known as Godwin Austin) in the Karakoram Range (It lies in the Karakoram range, partially in
the Gilgit-Baltistan region of Pakistan-administered Kashmir and partially in a China-administered territory Trans-Karakoram
Tract included in the Taxkorgan Tajik Autonomous County of Xinjiang.).
25. Kullu Valley is located between the Mountain ranges of Dhauladhar and Pir Panjal
26. The longest coastline (Andaman and Nicobar > Gujarat )
27. The number of coastal states in India is 9. See them on map.
28. Mainland coastal line is 6100 km and overall it is 7516.6 km (including the coastal line of the island of Andaman and Nicobar and
Lakshadweep).
ADVICE – YOU ARE ADVISED TO GO THROUGH THIS SECTION SELECTIVELY FROM THE UPSC AND STATE PSC BOOSTER BOOK available
freely on the website www.neeleshair442.com
You will get many additional information.

INDIA : DRAINAGE SYSTEM


CLASS XI: Physical Environment Chapter 3 (Drainage System)
CLASS IX Chapter 3 (Drainage)

Total Number of Questions Asked from 2011-22 12


My Instructions related to these Chapters This section is also very important. Know the origin, tributaries, rivers
originating from eastern and western ghats, difference between the
Himalayan and peninsular rivers, locations of rivers etc.
You are advised to look for ATLAS too while reading these chapters
beside looking for map given in NCERT

PREVIOUS YEAR QUESTIONS RELATED TO THESE CHAPTERS OR TOPICS


S.No. Question Answer Explanation
1 The Brahmaputra, Irrawady and Mekong rivers originate d The Brahmaputra River flows an odd channel to flow to the
in Tibet and flow through narrow and parallel mountain sea. It flows towards east for half of its length in mountains

JOIN ME AT TELEGRAM – UPSC PRELIMS WITH NEELESH (AIR 442 UPSC CSE 2021)
https://t.me/UPSCPrelimsWithNeelesh
VISIT OUR WEBSITE – neeleshair442.com ©Copyright reserved with author
ranges in their upper reaches. Of these rivers, and then takes a U-turn to enter plains in Assam. The place
Brahmaputra makes a “U” turn in its course to flow into where it takes a U-turn, is known as Great Bend of Yarlung
India. This “U” turn is due to: (2011) Tsangpo/Brahmaputra. At the same place the river forms the
a. Uplift of folded Himalayan series. deepest canyon of the world (Grand Canyon). The U-turn in
b. Syntaxial bending of geologically young Himalayas. Brahmaputra River is due to uplift of folded Himalayan series
c. Geo-tectonic disturbance in the tertiary folded and syntaxial bending of geologically young Himalayas.
mountain chains.
d. Both (a) and (b) above.

2 The Narmada River flows to the west, while most other a It is clearly mentioned in NCERT. The reason for the flow of
peninsular large rivers flow to the east. Why? (2013) the river Narmada from the east to west is that it flows
1. It occupies a linear rift valley. through a linear rift valley between the two mountain ranges
2. It flows between the Vindhyas and the Satpuras. ‘Vindhya’ and ‘Satpura’
3. The land slopes to the west from Central India.
Select the correct answer using the codes given below:
a. 1 only
b. 2 and 3
c. 1 and 3
d. None

3 Consider the following rivers: (2021) B • The Eastern Ghats are a discontinuous range of
1. Brahmani mountains along India’s eastern coast. The Eastern Ghats
2. Nagavali pass through Odisha, Andhra Pradesh to Tamil Nadu in the
3. Subarnarekha south passing some parts of Karnataka as well as Telangana.
4. Vamsadhara
Which of the above rise from the Eastern Ghat? • Rivers orginating on the Eastern Ghats include:
a. 1 and 2 ➢ Baitarani River
b. 2 and 4 ➢ Rushikulya River
c. 3 and 4 ➢ Palar River
d. 1 and 3 ➢ Champavathi River
➢ Sarada River

JOIN ME AT TELEGRAM – UPSC PRELIMS WITH NEELESH (AIR 442 UPSC CSE 2021)
https://t.me/UPSCPrelimsWithNeelesh
VISIT OUR WEBSITE – neeleshair442.com ©Copyright reserved with author
➢ Sileru River
➢ Gundlakamma River
➢ Swarnamushi
➢ Vellar River
➢ Budhabalanga River
➢ Vamsadhara River
➢ Nagavali River
➢ Gosthani River
➢ Sabari River
➢ Tammileru
➢ Pennai Yaru River
➢ Kundu River
➢ Penna River
Therefore, the correct answer is b

4 Consider the following rivers: (2014) b Five major rivers of Arunachal Pradesh are Kameng,
1. Barak Subansiri, Siang, Tirap and Lohit.
2. Lohit Barak does not flow through Arunachal Pradesh. It rises in
3. Subansiri the hill country of Manipur State. After Manipur it flows
Which of the above flows/ flow through Arunachal through Mizoram and then into Assam.
Pradesh?
a. 1 only
b. 2 and 3 only
c. 1 and 3 only
d. 1,2 and 3

5 With reference to the Indus River system, of the following d The answer is Sutlej River. Check the map
four rivers, three of them pour into one of them which
joins the Indus Direct. Among the following, which one is
such river that joins the Indus direct? (2021)
a. Chenab

JOIN ME AT TELEGRAM – UPSC PRELIMS WITH NEELESH (AIR 442 UPSC CSE 2021)
https://t.me/UPSCPrelimsWithNeelesh
VISIT OUR WEBSITE – neeleshair442.com ©Copyright reserved with author
b. Jhelum
c. Ravi
d. Sutlej

6 Consider the following rivers: (2015) d The Penganga, the Indravati, the Pranhita and the Manjra are
1. Vamsadhara the principal tributaries of Godavari river.
2. Indravati
3. Pranahita You are advised to prepare the important rivers and their
4. Pennar tributaries from the NCERTs and make a tabular listing and
Which of the above are the tributary of Godavari? keep revising it occassionally.
a. 1,2 and 3
b. 2,3 and 4
c. 1,2 and 4
d. 2 and 3 only

7 Which of the following is/are tributary/ tributaries of d The left bank tributaries of Brahmaputra are Dibang or
Brahmaputra? (2016) Sikang and Lohit. The right bank tributaries of Brahmaputra
1. Dibang are the Subansiri, Kameng, Manas and Sankosh.
2. Kameng
3. Lohit
Select the correct answer using the code given below:
a. 1 only
b. 2 and 3 only
c. 1 and 3 only
d. 1,2 and 3

8 With reference to river Teesta, consider the following b The Teesta river is 309 km long river flowing through the
statements: (2017) Indian State of West Bengal and Sikkim, before going to Bay
1. The source of river Teesta is the same as that of of Bengal through Bangladesh.
Brahmaputra but it flows through Sikkim. The Teesta River originates from the Pahunri (or Teesta
Kangse) glacier whereas Brahmaputra river originates in

JOIN ME AT TELEGRAM – UPSC PRELIMS WITH NEELESH (AIR 442 UPSC CSE 2021)
https://t.me/UPSCPrelimsWithNeelesh
VISIT OUR WEBSITE – neeleshair442.com ©Copyright reserved with author
2. River Rangeet originates in Sikkim and it is a Angsi glacier. Teesta is a tributary of river Brahmaputra.
tributary of river Teesta. Hence, 1 and 3 are not correct.
3. River Teesta flows into Bay of Bengal on the border
of India and Bangladesh. The Rangeet or Rangit is a tributary of the Teesta river,
Which of the statements given above is/are correct? which is the largest river in the Indian State of Sikkim. The
a. 1 and 3 only Rangeet river originates in the Himalayan mountains in the
b. 2 only West Sikkim district. The river also forms the boundary
c. 2 and 3 only between Sikkim and Darjeeling district. Hence only option 2
d. 1, 2 and 3 is correct

9 Recently, linking of which of the following rivers was b This was a current affairs question
undertaken? (2016) There are many interlinking projects in India. Out of them,
a. Cauvery and Tungabhadra till now only Godavari and Krishna has been completed.
b. Godavari and Krishna
c. Mahanadi and Sone
d. Narmada and Tapti

10 Consider the following pairs: (2019) a Bandar punch is a mountain massif of the Garhwal division
Glacier River of the Himalayas, in the Uttarakhand state. It is part of the
1. Bandarpunch : Yamuna Sankari Range and lies within the Govind Pashu Vihar
2. Bara Shigri : Chenab National Park and Sanctuary. It is a major watershed for the
3. Milam : Mandakini headwaters of the Yamuna River, whose source lies above
4. Siachen : Nubra Yamnotri, on the west end of the massif below White Peak.
5. Zemu : Manas Bara Shigri is the largest glacier is located in the state of
Which of the pairs given above are correctly matched? Himachal Pradesh. It feeds the Chenab river.
a. 1, 2 and 4 Milan glacier is a major glacier of Munsiyari, part of the
b. 1, 3 and 4 Pithoragarh district of Uttarakhand. The glacier is the source
c. 2 and 5 of Goriganga river and not of the Mandakini Glacier. Hence,
d. 3 and 5 pair 3 is not correct.
Siachen glacier is located in the eastern Karakoram range in
the Himalayas just northeast of the point NJ9842 where the

JOIN ME AT TELEGRAM – UPSC PRELIMS WITH NEELESH (AIR 442 UPSC CSE 2021)
https://t.me/UPSCPrelimsWithNeelesh
VISIT OUR WEBSITE – neeleshair442.com ©Copyright reserved with author
Line of Control between India and Pakistan ends. The
glacier’s melting waters are the main source of the Nubra
River in the Indian region of Ladakh which drains into the
Shyok River. The Shyok, in turn, joins Indus River which flows
through Pakistan. Thus, the glacier is a major source of the
Indus and feeds the largest irrigation system in the world.
Zemu glacier is the largest glacier in the Eastern Himalayas,
located at the base of Kangchenjunga in the Himalayan
region of Sikkim. The glacier is the source of water for
numerous rivers, as it feeds them when it melts. Main river
of them is the Teesta River and not Manas. Hence, pair 5 is
also not correctly matched
Advice – Prepare a list of important glaciers and the rivers
having their origin in it.

11 Consider the following pairs: (2019) a Pandharpur is a well known pilgrimage town on the banks of
Famous Place River Chandrabhaga river in the Solapur district of Maharashtra.
1. Pandharpur : Chandrabhaga The Vithoba temple here attracts about a million Hindu
2. Tiruchirapalli : Cauvery pilgrims during the major yatra in Ashadh month (June-July).
3. Hampi : Malaprabha Tiruchirappalli also known as Trichy, is a major tier II city in
Which of the pairs given above are correctly matched? Tamil Nadu. The two major rivers draining Trichy are the
a. 1 and 2 only Kaveri and its tributary the Kollidam, but the city is drained
b. 2 and 3 only by the Uyyakondan Channel, Koraiyar and Kudamurritti river
c. 1 and 3 only channels.
d. 1, 2 and 3 Hampi, a UNESCO world Heritage site in east Central
Karnataka, is situated on the banks of Tungabhadra River not
Malprabha. Hence, pair 3 is not correctly matched.

12 Gandikota canyon of South India was created by which c • Gandikota is small village in the Kadapa district of
one of the following rivers? (2022) Andhra Pradesh. The village is majorly known for housing
a. Cauvery

JOIN ME AT TELEGRAM – UPSC PRELIMS WITH NEELESH (AIR 442 UPSC CSE 2021)
https://t.me/UPSCPrelimsWithNeelesh
VISIT OUR WEBSITE – neeleshair442.com ©Copyright reserved with author
b. Manjira the spectacular gorge which is famously adjudged as the
c. Pennar Grand Canyon of India.
d. Tungabhadra • The stunning gorge has been created by the waters
of the famous river Pennar that streams from the Erramala
hills. It is also known as the Hidden Grand Canyon of India as
it isn’t very well known around the country.
Why in news?
• In Dec. 2021, Andhra Pradesh govt. said “In a bid to
improve tourism in the state, the AP government is going to
construct a ropeway to the village of Gandikota – popularly
known as the Grand Canyon of India”. So, this is perhaps the
reason for this question being asked in 2022 paper.

INDIA: CLIMATE
CLASS XI: Physical Environment Chapter 4 (Climate)
CLASS IX Chapter 4 (Climate)

Total Number of Questions Asked from 2011-22 4


My Instructions related to these Chapters This is mainly a conceptual chapter. Once you understand the
concept of Monsoon, this portion and its questions become very
easy. Focus on understanding the mechanism of Monsoon and El-
Nino, La-Nina, IOD, ENSO etc. You must not make a mistake in
concepts as they function as your lifeline for clearing the cut off of
UPSC Prelims. So, understand the mechanism of Monsoons properly.

JOIN ME AT TELEGRAM – UPSC PRELIMS WITH NEELESH (AIR 442 UPSC CSE 2021)
https://t.me/UPSCPrelimsWithNeelesh
VISIT OUR WEBSITE – neeleshair442.com ©Copyright reserved with author
PREVIOUS YEAR QUESTIONS RELATED TO THESE CHAPTERS OR TOPICS
S.No. Question Answer Explanation
1 Consider the following statements: (2012) c Northern plains receive rains by South-West monsoon.
1. The duration of the monsoon decreases from These monsoons, being the branch of Bay of Bengal
southern India to northern India. monsoon, move westwards along the Himalayas.
2. The amount of annual rainfall in the northern These monsoon winds on moving towards the western part,
plains of India decreases from east to west. gradually loose moisture causing the less rainfall.
Which of the statements given above is/are correct?
a. 1 only The duration of monsoon is maximum in the southern India,
b. 2 only while it keeps decreasing towards the northern India.
c. Both 1 and 2
d. Neither 1 nor 2

2 The seasonal reversal of winds is the typical c There is the reversal of wind in the Monsoon season. So,
characteristic of: (2014) answer is c. For more details, you must read your chapter on
(a) Equatorial climate Monsoon properly.
(b) Mediterranean climate
(c) Monsoon climate
(d) All of the above climates

3 La Nina is suspected to have caused recent floods in d El Nino refers to the large scale ocean-atmosphere climate
Australia. How is La Nina different from El Nino? interaction linked to periodic warming in sea surface
(2011) temperatures across the central and east-central Equatorial
1. La Nina is characterized by unusually cold Pacific. It is associated with high pressure in the western
ocean temperature in equatorial Indian Ocean Pacific. El Nino adversely impacts the Indian monsoons and
whereas El Nino is characterized by unusually warm hence, agriculture in India.
ocean temperature in the equatorial Pacific Ocean. The cool surface water off the Peruvian coast goes warm
2. El Nino had adverse effect on south-west because of El Nino. When the water is warm, the normal
monsoon of India, but La Nina has no effect on trade winds get lost or reverse their direction. Hence, the
monsoon climate. flow of moisture-laden winds is directed towards the coast
Which of the statement given above is/are correct? of Peru from the western Pacific (the region near northern
JOIN ME AT TELEGRAM – UPSC PRELIMS WITH NEELESH (AIR 442 UPSC CSE 2021)
https://t.me/UPSCPrelimsWithNeelesh
VISIT OUR WEBSITE – neeleshair442.com ©Copyright reserved with author
(a) 1 only Australia and South East Asia). This causes heavy rains in
(b) 2 only Peru during the El Nino years robbing the Indian
(c) Both 1 and 2 subcontinent of its normal monsoon rains. The larger the
(d) Neither 1 nor 2 temperature and pressure difference, the larger the rainfall
shortage in India.

La Nina means ‘little girl’ in Spanish and is also known as El


Viejo or ‘cold event’. Here, the water temperature in the
Eastern Pacific gets colder than normal. As a result of this,
there is a strong high pressure over the eastern equatorial
Pacific. Now, there is low pressure in the Western Pacific and
off Asia. La Nina causes drought in Peru and Ecuador, heavy
floods in Australia, high temperatures in Western Pacific,
Indian Ocean, off the Somalian coast and good monsoon
rains in India. A La Nina is actually beneficial for the Indian
monsoon.

4 With reference to ‘Indian Ocean Dipole (IOD)’ b The Indian Ocean Dipole (IOD), also known as the Indian
sometimes mentioned in the news while forecasting Nino, is an irregular oscillation of sea-surface temperatures
Indian monsoon, which of the following statements in which the western Indian Ocean becomes alternately
is/are correct? (2017) warmer and then colder than the eastern part of the ocean
1. IOD phenomenon is characterized by a (not tropical eastern Pacific Ocean). Hence, statement 1 is
difference in sea surface temperature between not correct.
tropical Western Indian Ocean and tropical Eastern The IOD also affects the strength of monsoons over the
Indian Ocean. Indian subcontinent. A significant positive IOD occurred in
2. An IOD phenomenon can influence an El 1997-98, with another in 2006. The IOD is one aspect of the
Nino’s impact on the monsoon. general cycle of global climate, interacting with similar
Select the correct answer using the code given phenomena like the EI Nino-Southern Oscillation (ENSO)in
below: the Pacific Ocean. IOD has a much more significant effect on
a. 1 only the rainfall patterns in South-East Australia than the EI-Nino
b. 2 only

JOIN ME AT TELEGRAM – UPSC PRELIMS WITH NEELESH (AIR 442 UPSC CSE 2021)
https://t.me/UPSCPrelimsWithNeelesh
VISIT OUR WEBSITE – neeleshair442.com ©Copyright reserved with author
c. Both 1 and 2 Southern Oscillation in the Pacific Ocean. Hence, statement
d. Neither 1 nor 2 2 is correct.

INDIA: NATURAL VEGETATION


CLASS XI: Physical Environment Chapter 5 (Natural Vegetation)
CLASS X Chapter 2 (Forest and Wildlife Resources)
CLASS IX Chapter 5 (Natural Vegetation and Wildlife)
CLASS VI Chapter 8 (Climate, Vegetation and Wildlife)

Total Number of Questions Asked from 2011-22 12


My Instructions related to these Chapters This is also one of the very important sections. You should know the
vegetation types, their location, their characteristics, and their flora
(Vegetation) and fauna (Wildlife).
In NCERT, National Park/Wildlife Sanctuary etc. portion is not given
properly. So, it needs to be prepared separately.

PREVIOUS YEAR QUESTIONS RELATED TO THESE CHAPTERS OR TOPICS


S.No. Question Answer Explanation
1 In India, in which one of the following types of forests is a Tropical moist deciduous forests are one of the most
teak a dominant tree species? (2015) common forests in India. They're also known as monsoon
a. Tropical moist deciduous forest. forests. Except in the western and northwestern parts of
b. Tropical rain forest. India, moist deciduous forests may be found practically
c. Tropical thorn scrub forest. everywhere.
d. Temperate forest with grasslands. The tree species found in these forests are teak, sal, laurel,
rosewood, amla, jamun, bamboo, neem, peepal etc.

2 If a tropical rain forest is removed, it does not a Despite the amount of vegetation in the tropical rain forest,
regenerate quickly as compared to a tropical deciduous the soil contains less organic matter than that of temperate
forest. This is because: (2011)
JOIN ME AT TELEGRAM – UPSC PRELIMS WITH NEELESH (AIR 442 UPSC CSE 2021)
https://t.me/UPSCPrelimsWithNeelesh
VISIT OUR WEBSITE – neeleshair442.com ©Copyright reserved with author
a) The soil of rain forest is deficient in nutrients forests, because the warm humid conditions encourage
b) Propagules of the trees in a rain forest have poor faster decay and recycling of nutrients back into living forest.
viability Thus, the tropical rain forest does not regenerate quickly as
c) The rain forest species are slow growing compared to a tropical deciduous forest.
d) Exotic species invade the fertile soil of rain forest

3 If you travel through the Himalayas, you are likely to see a Sandalwood is an evergreen tree which generally grows in
which of the following plants naturally growing there? the dry, deciduous forests of the Deccan Plateau. It grows in
(2014) Karnataka, Kerala, Tamil Nadu and Andhra Pradesh.
1. Oak
2. Rhododendron So, only 1 and 2 is correct.
3. Sandalwood
Select the correct answer using the code given below:
a. 1 and 2 only
b. 3 only
c. 1 and 3 only
d. 1,2 and 3

4 Which one of the following regions of India has a d The South Andaman forests have a profuse growth of
combination of mangrove forest, evergreen forest and epiphytic vegetation, mostly ferns and orchids. The Middle
deciduous forest? (2015) Andamans harbours mostly moist deciduous forests. North
a. North coastal Andhra Pradesh Andamans is characterized by the wet evergreen type, with
b. South- West Bengal plenty of woody climbers. Grasslands occur only in the
c. Southern Saurashtra Nicobars, and while deciduous forests are common in the
d. Andaman and Nicobar Islands Andamans, they are almost absent in the Nicobars.
The typical forest coverage of Andaman and Nicobar Islands
is made- up of twelve types namely (1) Giant evergreen
forest (2) Andamans tropical evergreen forest (3) Southern
hilltop tropical evergreen forest (4) Cane brakes (5) Wet
bamboo brakes (6) Andamans semi-evergreen forest (7)
Andamans moist deciduous forest (8) Andamans secondary

JOIN ME AT TELEGRAM – UPSC PRELIMS WITH NEELESH (AIR 442 UPSC CSE 2021)
https://t.me/UPSCPrelimsWithNeelesh
VISIT OUR WEBSITE – neeleshair442.com ©Copyright reserved with author
moist deciduous forest (9) Littoral Forest (10) Mangrove
Forest (11) Brackish water mixed forest (12) Submontane hill
valley swamp forest.

5 With reference to ‘Red Sanders’, sometimes seen in a Pterocarpus santalinus, with the common names red
news, consider the following statements: (2016) sanders, red sandalwood, and Saunders wood, is a species of
1. It is a tree species found in a part of South India. Pterocarpus endemic to the southern Eastern Ghats
2. It is one of the most important trees in the Mountain range of South India. This tree is valued for the rich
tropical rain forest areas of South India. red color of its wood. The wood is not aromatic.
Which of the statements given above is/are correct? It was in national media during April 2015, when Andhra
a. 1 only Pradesh police killed 20 Red Sander smugglers in an
b. 2 only encounter. So, first statement is right.
c. Both 1 and 2 Red Sanders is associated with Tropical dry deciduous forest,
d. Neither 1 nor 2 as per environment ministry website, Hence, 2nd statement
is wrong.

6 Among the following States, which one has the most b Arunachal Pradesh has also been termed as ‘Orchid Paradise
suitable climatic conditions for the cultivation of a large of India’ because of the maximum concentration of orchid
variety of orchids with minimum cost of production, and species in the state. It has the most suitable climatic
can develop an export-oriented industry in this field? condition for the cultivation of variety of orchids
(2011)
a. Andhra Pradesh
b. Arunachal Pradesh
c. Madhya Pradesh
d. Uttar Pradesh

7 Consider the following States: (2015) c Refer your original source NCERT to know the spread of
1. Arunachal Pradesh Tropical Wet Evergreen Forest
2. Himachal Pradesh
3. Mizoram

JOIN ME AT TELEGRAM – UPSC PRELIMS WITH NEELESH (AIR 442 UPSC CSE 2021)
https://t.me/UPSCPrelimsWithNeelesh
VISIT OUR WEBSITE – neeleshair442.com ©Copyright reserved with author
In which of the above states do ‘Tropical Wet Evergreen
Forests’ occur? (2015)
a. 1 only
b. 2 and 3 only
c. 1 and 3 only
d. 1,2 and 3

8 Consider the following pairs: (2013) d - Jim Corbett National Park is located in Nainital district
National Park River flowing through of Uttarakhand. River flowing through the park is Ramganga.
the park - Kaziranga National Park is situated in Assam. The
1. Corbett National Park : Ganga Diphlu river flows through it.
2. Kaziranga National Park : Manas - Silent Valley national park is located in Kerala, in
3. Silent valley national park : Kaveri Nilgiri hills. Kunthipuzha river flows through it.
Which of the above pairs is/are correctly matched?
a. 1 and 2 Protective Area Network has an overlapping portion with
b. 3 only environment with the environment. You are advised to club
c. 1 and 3 this part of Geography with environment for consolidated
d. None preparation.

9 Consider the following pairs: (2013) a - The Nokrek Biosphere Reserve is located in the
1. Nokrek Biosphere Reserve : Garo Hills northeast of India on the Tura Range, which forms part of the
2. Logtak(Loktak) Lake : Barail range Meghalaya Plateau. The entire area is mountainous and
3. Namdapha National Park : Dafla Hills Nokrek is the highest peak of the Garo hills.
Which of the above pairs is/are correctly matched? - Loktak Lake, the largest freshwater lake in the
a. 1 only northeast India, also called the only Floating Lake in the
b. 2 and 3 only world due to the floating phumdis (heterogenous mass of
c. 1,2 and 3 vegetation, soil and organic matters at various stages of
d. None decomposition) on it, is located near Moirang in Manipur
state, India.
- Namdapha National Park is the largest protected area
in the Eastern Himalaya biodiversity hotspot and is located in

JOIN ME AT TELEGRAM – UPSC PRELIMS WITH NEELESH (AIR 442 UPSC CSE 2021)
https://t.me/UPSCPrelimsWithNeelesh
VISIT OUR WEBSITE – neeleshair442.com ©Copyright reserved with author
Arunachal Pradesh in Northeast India. It is also the third
largest national park in India in terms of area. It is located
between the Dapha Bum range of the Mishmi Hills and the
Patkai range

10 Consider the following pairs: (2014) c Gumti Wildlife Sanctuary is situated in Tripura.
1. Dampa Tiger Reserve : Mizoram Else both are correct.
2. Gumti Wildlife Sanctuary : Sikkim
3. Saramati Peak : Nagaland Go through the list of Biosphere Reserves, wildlife
Which of the above pairs is/are correct matched? sanctuaries, national parks, tiger reserves etc. in detail, as
a. 1 only these questions are asked often. So, work on those parts
b. 2 and 3 only which are asked frequently.
c. 1 and 3 only
d. 1, 2 and 3

11 Which of the following have coral reefs? (2014) a Coral reefs are the marine ecosystem, which are generally
1. Andaman and Nicobar Islands found in the equatorial water. They are held together by
2. Gulf of Kachchh calcium carbonate secreted by corals. Coral reefs are built by
3. Gulf of Mannar the colonies of tiny animals found in marine waters.
4. Sunder bans Coral reefs are not found in the Sunderbans.
Select the correct answer using the code given below: The Coral reefs in India are mainly restricted to the Andaman
a. 1,2 and 3 only and Nicobar Islands, Gulf of Mannar, Gulf of Kutch, Palk
b. 2 and 4 only Strait and the Lakshadweep islands. All of these reefs
c. 1 and 3 only are Fringing reefs, except Lakshadweep which are Atolls.
d. 1,2,3 and 4 There are Patchy corals present along the inter-tidal areas of
the central west coast like the intertidal regions
of Ratnagiri, Gaveshani Bank etc. The Hermatypic corals are
also present along the sea shore from Kollam in Kerala
to Enayam Puthenthurai in Tamil Nadu.

JOIN ME AT TELEGRAM – UPSC PRELIMS WITH NEELESH (AIR 442 UPSC CSE 2021)
https://t.me/UPSCPrelimsWithNeelesh
VISIT OUR WEBSITE – neeleshair442.com ©Copyright reserved with author
Please note that – coral reef is extremely important. Its
location must be known at the India level and also at the
world level for atleast major distributions

12 Which one of the following National Parks has a climate d Namdapha National Park is the largest protected area in the
that varies from tropical to subtropical, temperate and Eastern Himalaya biodiversity hotspot and is located in
arctic? (2015) Arunachal Pradesh in Northeast India. It is located in Eastern
a. Khangchendzonga National Park Himalayan sub-region and is recognized as one of the richest
b. Nandadevi National Park areas in biodiversity in India. Its climate varies from tropical
c. Neora Valley National Park to subtropical, temperate and arctic.
d. Namdapha National Park

INDIA: SOILS
CLASS XI: Physical Environment Chapter 6 (Soils)
Total Number of Questions Asked from 2011-22 3
My Instructions related to these Chapters The questions are occasionally asked but this section can give you
sure marks. As very limited types of questions can be made from it.
Focus on spread, properties, soil degradation reasons, and
conservation methods.

PREVIOUS YEAR QUESTIONS RELATED TO THESE CHAPTERS OR TOPICS


S.No. Question Answer Explanation
1 Which of the following statements regarding laterite soils c Laterite soil is reddish to yellow in color with a lower
of India are correct? (2013) content of potassium, phosphorus, nitrogen, lime, and
1. They are generally red in colour. magnesia. They are rich in iron and aluminium, and are
2. They are rich in nitrogen and potash. found in hot and wet tropical areas.
3. They are well developed in Rajasthan and UP. They are found in Karnataka, Tamil Nadu, Kerala, Assam,
4. Tapioca and cashew nuts grow well on these soils. Maharashtra, Odisha, West Bengal, Rajmahal hills, etc.

JOIN ME AT TELEGRAM – UPSC PRELIMS WITH NEELESH (AIR 442 UPSC CSE 2021)
https://t.me/UPSCPrelimsWithNeelesh
VISIT OUR WEBSITE – neeleshair442.com ©Copyright reserved with author
Select the correct answer using the codes given below: Predominant crops of laterite soils are coffee, rubber,
a. 1,2 and 3 cashewnut and tapioca.
b. 2,3 and 4
c. 1 and 4 Instruction - Prepare soil spread and their properties
d. 2 and 3 only from your NCERT. It is very important.

2 In India, the problem of soil erosion is associated with b Soil erosion is mainly caused by the deforestation, as
which of the following? (2014) they tend to remove the protective layer of vegetation
1. Terrace cultivation and tend to make soil lose.
2. Deforestation Terrace cultivation is a method of farming on the hill
3. Tropical Climate slopes by making terraces. This practice reduces the soil
Select the correct answer using the code given below? erosion.
a. 1 and 2 only There is less soil erosion in tropical areas as compared
b. 2 only to sub-tropical areas.
c. 1 and 3 only
d. 1,2 and 3

3 Contour bunding is a method of soil conservation used in: d Contour bunding, involves the placement of lines of
(2013) stones along the natural rises of a landscape. It helps to
(a) Desert margins, liable to strong wind action. capture and hold rainfall before it can become runoff. It
(b) Low flat plains, close to stream courses, liable to also inhibits wind erosion by keeping the soil heavy and
flooding. moist.
(c) Scrublands, liable to spread to weed growth. Contour bunding is associated with terracing, to check
(d) None of the above. the flow of water on a hill slope in order to reduce soil
erosion.
It is mainly done on the hill slopes and uphill regions.

JOIN ME AT TELEGRAM – UPSC PRELIMS WITH NEELESH (AIR 442 UPSC CSE 2021)
https://t.me/UPSCPrelimsWithNeelesh
VISIT OUR WEBSITE – neeleshair442.com ©Copyright reserved with author
INDIA: NATURAL HAZARDS AND DISASTER
CLASS XI: Physical Environment Chapter 7 (Natural Hazards and
Disaster
Total Number of Questions Asked from 2011-22 0
My Instructions related to these Chapters No question has been asked from here in the last ten years. However,
focus on mechanism/structure/prone areas of earthquakes, tsunami,
tropical cyclone, floods, droughts and landslide.
Instead of recalling word by word, try to understand the mechanisms
and spread as per location factors. Do not try to remember word by
word only to get confused. Instead develop an understanding.
However, whenever there is any major disaster, you must keep note
of that as it may be asked.

INDIA: POPULATION, MIGRATION, HUMAN DEVELOPMENT, HUMAN SETTLEMENT


CLASS XII:
India People and Chapter 1(Population) + Chapter 2
Economy (Migration) + Chapter 3 (Human
Development) + Chapter 4 (Human
Settlement))
CLASS IX (Chapter 6 (Population)
Total Number of Questions Asked from 2011-22 0
My Instructions related to these Chapters Though no question has been asked from these chapters, still few
portions remain important. Focus on population chapter. Read rest
of the chapter with understanding. There is not much data to
remember instead if you develop the understanding, you can easily
handle the questions if ever asked

JOIN ME AT TELEGRAM – UPSC PRELIMS WITH NEELESH (AIR 442 UPSC CSE 2021)
https://t.me/UPSCPrelimsWithNeelesh
VISIT OUR WEBSITE – neeleshair442.com ©Copyright reserved with author
INDIA: LAND RESOURCES AND AGRICULTURE
CLASS XII: India People and Chapter 5 (Land Resources and
Economy Agriculture)
CLASS X Chapter 1(Resources and
Development) – Note – overlapping
chapter with Soil +
Chapter 4 (Agriculture)
Total Number of Questions Asked from 2011-22 12
My Instructions related to these Chapters This section is very important. Agriculture is always the favorite topic of UPSC.
You must read the chapter in great detail. There is some outdated data. That
can be supplemented from the Economic Survey data and from other sites.
Rest, even the outdated date will give you the idea on many aspects. Look for
which are rainfed crops, which are kharif/rabi, what are the climatic conditions
of growth of crops etc. Keep reading PYQ while reading these chapters.

PREVIOUS YEAR QUESTIONS RELATED TO THESE CHAPTERS OR TOPICS


S.No. Question Answer Explanation
1 Consider the following crops in India: (2012) a Cowpea has a high content of protein and can be used as a
1. Cowpea nutritious feed for the livestock. It is used as green manure
2. Green gram due to its nitrogen fixing properties. Green gram is also very
3. Pigeon pea nutritious due to its high protein content and can be used as
Which of the above is/are used as pulse, fodder and green a green manure in many African countries.
manure?
a. 1 and 2 only Pigeon pea is mainly eaten in the form of split pulse as ‘dal’.
b. 2 only The use of pigeon pea as green manure crop is not very
c. 1 and 3 only common.
d. 1,2 and 3 The answer is (a) as per UPSC Official Answer Key.

JOIN ME AT TELEGRAM – UPSC PRELIMS WITH NEELESH (AIR 442 UPSC CSE 2021)
https://t.me/UPSCPrelimsWithNeelesh
VISIT OUR WEBSITE – neeleshair442.com ©Copyright reserved with author
2 Consider the following crops of India: (2012) d All the three crops i.e., Groundnut, sesamum and pearl
1. Groundnut millet are the rainfed crops
2. Sesamum
3. Pearl millet
Which of the above is/are predominantly rainfed crops?
a. 1 and 2 only
b. 2 and 3 only
c. 3 only
d. 1,2 and 3

3 Consider the following pairs: (2014) d Areca nut is produced in Karnataka, Tamil Nadu, Kerala, West
Region Well-known for the production of Bengal, Assam, etc.
1. Kinnaur : Areca nut Uttar Pradesh is the leading state in the production of Mango
2. Mewat : Mango while Mewat is in Haryana.
3. Coromandel : Soya Bean Soyabean is primarily grown in Madhya Pradesh while
Which of the above pairs is/ are correctly matched? coromandel coast is in the south east coast region of Indian
a. 1 and 2 only Subcontinent.
b. 3 only
c. 1,2 and 3
d. None

4 Consider the following crops: (2013) c The Kharif cropping season starts with the onset of the
1. Cotton Indian subcontinent’s monsoon. Kharif crops are typically
2. Groundnut sown at the beginning of the first monsoon rains (depending
3. Rice on region to region). Harvesting season begins from the 3rd
4. Wheat week of September to October (the exact harvesting dates
Which of these are Kharif crops? differ from region to region).
(a) 1 and 4
(b) 2 and 3 only Kharif crops require good rainfall. The output of these crops
(c) 1, 2 and 3 depends upon the time and amount of rainwater. Paddy,

JOIN ME AT TELEGRAM – UPSC PRELIMS WITH NEELESH (AIR 442 UPSC CSE 2021)
https://t.me/UPSCPrelimsWithNeelesh
VISIT OUR WEBSITE – neeleshair442.com ©Copyright reserved with author
(d) 2, 3 and 4 maize, bajra, jowar, shorghum, soyabean, cotton, groundnut
are a few of the Kharif crops grown in India.

Rabi crops are known as winter crops. They are sown in


October or November. The crops are then harvested in
spring. These crops require frequent irrigation because they
are grown in dry areas. Wheat, gram, barley, oats, pulses,
mustard, linseed are some of the rabi crops grown in India.

5 The lower Gangetic plain is characterized by humid c Paddy is the main crop of the lower Gangetic plain. It
climate with high temperature throughout the year. requires an arable field, with humid climate and the high
Which one among the following pairs of crops is most temperature.
suitable for this region? (2011) Jute needs a plain alluvial soil and standing water. The
a. Paddy and cotton suitable climate for growing jute is warm and wet climate.
b. Wheat and Jute
c. Paddy and Jute
d. Wheat and cotton

6 Consider the following statements: (2014) d Cornstarch is made from maize kernels, which are high in
1. Maize can be used for the production of starch. starch, and used in the thickening agents of soup.
2. Oil extracted from maize can be feedstock for Maize has oil extracts that can be converted to ethanol or
biodiesel. biodiesel.
3. Alcoholic beverages can be produced by using An alcoholic drink is a drink that contains ethanol, common
maize. sources are maize and other grains.
Which of the statements given above is/are correct?
(a) 1 only
(b) 1 and 2 only
(c) 2 and 3 only
(d) 1, 2 and 3

JOIN ME AT TELEGRAM – UPSC PRELIMS WITH NEELESH (AIR 442 UPSC CSE 2021)
https://t.me/UPSCPrelimsWithNeelesh
VISIT OUR WEBSITE – neeleshair442.com ©Copyright reserved with author
7 Why does the Government of India promote the use of b Neem has proven Nitrification inhibition properties and
‘Neem-coated Urea’ in agriculture? (2016) hence slows down the release of nitrogen from urea and
a. Release of neem oil in the soil increases nitrogen makes available nitrogen over a longer period with minimum
fixation by the soil microorganisms. loss of nitrogen thereby increasing nitrogen use efficiency.
b. Neem coating slows down the rate of dissolution Hence, government of India promote the use of ‘Neem
of urea in the soil. coated urea’ in agriculture
c. Nitrous oxide, which is a greenhouse gas, is not at
all released into atmosphere by crop fields.
d. It is combination of a weedicide and a fertilizer for
particular crops.

8 What are the significances of a practical approach to b Sustainable Sugarcane Initiative (SSI) is a method of
sugarcane production known as ‘Sustainable Sugarcane sugarcane production which involves using less seeds, less
Initiative’? (2014) water and optimum utilization of fertilizers and land to
1. Seed cost is very low in this compared to the achieve more yields. Driven by farmers, SSI is an alternative
conventional method of cultivation. to the conventional seed, water and space intensive
2. Drip irrigation can be practiced very effectively in sugarcane cultivation.
this. SSI supports intercropping in sugarcane with crops like
3. There is no application of chemical/ inorganic wheat, potato, cowpea, French beans, chickpea, etc. In
fertilizers at all in this. addition to effective utilisation of land, this practice will
4. The scope of intercropping is more in this reduce the weed growth and give extra income to farmers.
compared to the conventional method of cultivation. The productivity of cane under SSI can be enhanced by
Select the correct answer using the code given below: practising drip irrigation with fertigation. Based on the soil
(a) 1 and 3 only type, drip irrigation can be scheduled daily or once in three
(b) 1, 2 and 4 only days.
(c) 2, 3 and 4 only
(d) 1, 2, 3 and 4

9 Which of the following is/are the advantage/advantages c In drip irrigation, water is applied near the plant root
of practicing drip irrigation? (2016) through emitters or drippers, on or below the soil surface, at
1. Reduction in weed. a low rate varying from 2-20 liters per hour. The soil moisture

JOIN ME AT TELEGRAM – UPSC PRELIMS WITH NEELESH (AIR 442 UPSC CSE 2021)
https://t.me/UPSCPrelimsWithNeelesh
VISIT OUR WEBSITE – neeleshair442.com ©Copyright reserved with author
2. Reduction in soil salinity. is kept at an optimum level with frequent irrigations. Among
3. Reduction in soil erosion. all irrigation methods, drip irrigation is the most efficient and
can be practiced for a large variety of crops, especially in
Select the correct answer using the code given below: vegetables, orchard crops, flowers and plantation crops.
(a) 1 and 2 only Merits:
(b) 3 only Fertilizer and nutrient loss are minimized due to localized
(c) 1 and 3 only application and reduced leaching. Field leveling is not
(d) None of the above is an advantage of practicing necessary. Recycled non-potable water can be used. Water
drip irrigation application efficiency increases. Soil erosion and weed
growth is lessened etc.
However, the primary manmade cause of soil salinity is
irrigation. Rainwater or groundwater used in irrigation
contains salts, which remain behind in the soil after the
water has evaporated.

10 With reference to the cultivation of Kharif crop in India in a As per the Economic Survey 2022-23, the area under the rice
the last five years, consider the following statements: cultivation remains the highest in India amongst the food
(2019) crops. So, statement 1 is correct.
1. Area under rice cultivation is the highest. Area under the cultivation of Jowar in 2021-22 is 1.5 million
2. Area under the cultivation of jowar is more than hectares in kharif season while for the oilseeds it is 19.8
that of oilseeds. million hectares in Kharif season. So, statement 2 is wrong.
3. Area of cotton cultivation is more than that of Area under cotton cultivation is 11.9 million hectare while
sugarcane. for sugarcane it is 5.1 million hectare. So, statement 3 is
4. Area under sugarcane cultivation has steadily correct.
decreased. Area under the sugarcane has been fluctuating in the last
Which of the statements given above are correct? decade. So, statement 4 is not correct
a. 1 and 3 only
b. 2, 3 and 4 only
c. 2 and 4 only
d. 1, 2, 3 and 4

JOIN ME AT TELEGRAM – UPSC PRELIMS WITH NEELESH (AIR 442 UPSC CSE 2021)
https://t.me/UPSCPrelimsWithNeelesh
VISIT OUR WEBSITE – neeleshair442.com ©Copyright reserved with author
11 Consider the following states: (2022) c • India is one of the world’s largest tea producers,
1. Andhra Pradesh though major of its tea produced is consumed within India
2. Kerala itself. Most popular teas, such as Assam and Darjeeling, also
3. Himachal Pradesh grow only in India.
4. Tripura • The major tea producing provinces in India are:
How many of the above are generally known as tea- Assam, West Bengal, Tamil Nadu, Kerala, Tripura, Arunachal
producing States? Pradesh, Himachal Pradesh, Karnataka, Sikkim, Nagaland,
a. Only one state Uttarakhand, Manipur, Mizoram, Meghalaya, Bihar, Odisha.
b. Only two states • The Kangra valley of Himachal Pradesh is one of the
c. Only three states most important sites for the production of tea. Due to the
d. All four states flavour and aroma of the tea, it is also known as the ‘Valley
of gods; The green and black teas are the two very
prominent varieties of Himachal Pradesh.
• Munnar Tea Plantation, Kerala: In the mountain
ranges of the western, Munnar is the hill station. Munnar tea
plantation is located at a height of 5,200 feet above the level
of the sea.
• Tripura is categorized as a traditional tea-growing
State.
Hence, (c) is the correct answer.

12 With reference to the usefulness of the by-products of c Bagasse is often used as a primary fuel source for sugar mills;
sugar industry, which of the following statements is/are when burned in quantity, it produces sufficient heat energy
correct? (2013) to supply all the needs of a typical sugar mill, with energy to
1. Bagasse can be used a biomass fuel for the spare. So, statement 1 is correct.
generation of energy.
2. Molasses can be used as one of the feedstocks for Statement 2 is not correct. Since, molasses is not used for
the production of synthetic chemical fertilizers. the production of synthetic chemical fertilizers.
3. Molasses can be used for the production of
ethanol.
Select the correct answer using the codes given below:

JOIN ME AT TELEGRAM – UPSC PRELIMS WITH NEELESH (AIR 442 UPSC CSE 2021)
https://t.me/UPSCPrelimsWithNeelesh
VISIT OUR WEBSITE – neeleshair442.com ©Copyright reserved with author
(a) 1 only Molasses can be used for the production of ethanol. 1 ton of
(b) 2 and 3 only molasses (sugar residue) produces nearly 240 liters of
(c) 1 and 3 only ethanol. So, statement 3 is correct.
(d) 1, 2 and 3

INDIA: WATER RESOURCES


CLASS XII: India People and Chapter 6 (Water Resources)
Economy
CLASS X Chapter 3 (Water Resources
Total Number of Questions Asked from 2011-22 4
My Instructions related to these Chapters This section has limited importance. Focus on watershed
management, rainwater harvesting, multipurpose river project’s
location, dams and the rivers, Meghalaya living bridge
PREVIOUS YEAR QUESTIONS RELATED TO THESE CHAPTERS OR TOPICS
S.No. Question Answer Explanation
1 What is common to the places known as Aliyar, Isa pur d All the place mentioned in the question are water reservoirs
and Kangsabati? (2019) of national importance.
a. Recently discovered uranium deposits.
b. Tropical rain forests.
c. Underground cave systems.
d. Water reservoirs.

2 Consider the following pairs: (2022) c • Ghataprabha reservoir is built near Ghataprabha,
Reservoirs States Karnataka across the Ghataprabha River, to store water for
1. Ghataprabha : Telangana irrigation. The Ghataprabha river is a major tributary of the
2. Gandhi Sagar : Madhya Pradesh river Krishna.
3. Indira Sagar : Andhra Pradesh • The Gandhi Sagar Dam is one of the four major dams
4. Maithon : Chhattisgarh built on India’s Chambal River. The dam is located in the
How many pairs given above are not correctly matched? Mandsaur district of the state of Madhya Pradesh.

JOIN ME AT TELEGRAM – UPSC PRELIMS WITH NEELESH (AIR 442 UPSC CSE 2021)
https://t.me/UPSCPrelimsWithNeelesh
VISIT OUR WEBSITE – neeleshair442.com ©Copyright reserved with author
CIVIL SERVICES WITH NEELESH
(TELEGRAM CHANNEL – UPSC PRELIMS WITH NEELESH)
(YOUTUBE CHANNEL – CIVIL SERVICES WITH NEELESH)
Note – All our initiatives (both free and paid) are on these channels only.

OUR INITIATIVES (Running Currently)

1. PRELIMS MENTORSHIP 2024 (BOTH FREE AND PAID)


2. ETHICS AND ESSAY EVALUATION – ER NEELESH AIR 442 UPSC_CSE2021 (https://t.me/UpscWithNeelesh_AIR442)
3. SOCIOLOGY FRAMEWORK - TELEGRAM – SOCIOLOGY WITH NEELESH (https://t.me/SociologyWithNeelesh)
4. FREE CSAT PROGRAM – ON YOUTUBE CHANNEL – CIVIL SERVICES WITH NEELESH
5. INTEGRATED MENTORSHIP FOR 2025 (PRELIMS AND MAINS) – YEARLONG MENTORSHIP
6. SHORT NOTES FOR ALL SUBJECTS (HANDWRITTEN)

UPCOMING INITIATIVES

1.PRELIMS TEST SERIES FOR 2025


2. MAINS TEST SERIES FOR 2025
3. ESSAY VIDEO BATCH FOR 2024 AND 2025
4. ETHICS VIDEO BATCH FOR 2024 AND 2025
5. SOCIOLOGY VIDEO BATCH FOR 2025 BASED ON PYQ
6. CSAT VIDEO BATCH
7. MAINS MARATHON 2024 AND MUCH MORE
a. Only one pair • The Indira Sagar Dam is the largest dam in India, in
b. Only two pairs terms of volume of water stored in the reservoir. It is located
c. Only three pairs on the Narmada River at the town of Narmada Nagar, Punasa
d. All four pairs in the Khandwa district of Madhya Pradesh.
• Maithon dam is located on the banks of river Barakar.
It is located about 48 kms from the Coal city of Dhanbad,
Jharkhand.
So, three pairs are not correctly matched.

3 What are the benefits of implementing the ‘Integrated c - Linking of country’s perennial rivers with seasonal
Watershed Development Programme? (2014) rivers is not the target of the Integrated Watershed
1. Prevention of soil runoff. Development Programme.
2. Linking the country’s perennial rivers with Integrated Watershed Development Programme
seasonal rivers. The main objectives of the IWMP are to restore the ecology
3. Rainwater harvesting and recharge of balance by harnessing, conserving and developing degraded
groundwater table. natural resources such as soil, vegetative cover and water. The
4. Regeneration of natural vegetation. outcomes are prevention of soil run off, regeneration of
Select the correct answer using the code given below: natural vegetation, rain water harvesting and recharging of
(a) 1 and 2 only ground water table. This enables multi-cropping and the
(b) 2, 3 and 4 only introduction of diverse agro based activities, which help to
(c) 1, 3 and 4 only provide sustainable, livelihoods to the people.
(d) 1, 2, 3 and 4
The Central and State share for the Integrated Watershed
Development projects is in the ratio of 90:10.
The main aims of this programme are as follows:
• To restore the ecological balance by harnessing, conserving
and developing degraded natural resources such as soil,
vegetative cover and water.
• The outcomes are prevention of soil run-off, soil loss.
• Regeneration of natural vegetation.

JOIN ME AT TELEGRAM – UPSC PRELIMS WITH NEELESH (AIR 442 UPSC CSE 2021)
https://t.me/UPSCPrelimsWithNeelesh
VISIT OUR WEBSITE – neeleshair442.com ©Copyright reserved with author
• Rain water harvesting and recharging of the ground Water
table.
• Enabling multi-cropping and the introduction of diverse
agro-based activities, which help to provide sustainable
livelihoods to the people residing in the watershed area.

The main objectives of this programme are as follows:


• To dissipate soil and water erosion and surface run-off.
• To harvest/recycle surface runoff and rainwater.
• To enhance soil moisture regime/water holding capacity.
• To promote sub-surface flow, base flow and ground water
recharge.
• To improve soil health and tilth.
• To improve production and productivity.
•To promote generation and gainful employment
opportunities.

4 ln a particular region in India, the local people train the a Meghalaya is known for its dense forests and living root
roots of living trees into robust bridges across the bridges that are believed to be built by the tribes of Khasi (a
streams. As the time passes, these bridges become tribe living in the eastern parts of Meghalaya). The structure
stronger. These unique living root bridges are found in: of the bridge is entirely made from tree branches, trunks, and
(2015) roots.This is a pedestrian bridge made of the roots of living
(a) Meghalaya trees - mainly used for crossing rivers.
(b) Himachal Pradesh Meghalaya's living root bridges were included as a place on
(c) Jharkhand the tentative list of UNESCO World Heritage Sites. Hence,
(d) Tamil Nadu option (a) is the correct answer.

JOIN ME AT TELEGRAM – UPSC PRELIMS WITH NEELESH (AIR 442 UPSC CSE 2021)
https://t.me/UPSCPrelimsWithNeelesh
VISIT OUR WEBSITE – neeleshair442.com ©Copyright reserved with author
INDIA: MINERAL AND ENERGY RESOURCES
Class XII: India People and Economy Chapter 7 (Mineral and Energy
Resources)
Class X Chapter 5 (Mineral and Energy
Resources)
Total Number of Questions Asked from 2011-22 7
My Instructions related to these Chapters This chapter is important. Learn the distribution etc of minerals
specially iron ore and energy resources – coal, petroleum, natural
gas, nuclear gas, solar energy and other forms of energy. This chapter
remains fully important.

PREVIOUS YEAR QUESTIONS RELATED TO THESE CHAPTERS OR TOPICS


S.No Question Answer Explanation
1 Consider the following minerals: (2020) d As per India Mineral Year Book 2021 as given on the official website of
1. Bentonite Indian Bureau of Mines, Minor minerals include barytes, bentonite,
2. Chromite calcite, corundum and sapphire, diaspore, dolomite, dunite & pyroxenite,
3. Kyanite felspar, fireclay, fuller’s earth, granite, gypsum and selenite, kaolin, ball
4. Sillimanite clay, other clays and shale, laterite, marble, mica, ochre, pyrophyllite,
In India, which of the above is/are officially quartz and other silica minerals, slate, sandstone, talc, soapstone and
designated as major minerals? steatite (Remember this list. After it, rest all minerals are major minerals).
a. 1 and 2 only
b. 4 only Rest are the major minerals.
c. 1 and 3 only
d. 2, 3 and 4 only

2 Which of the following is/are the a Majority of coal found in India has high ash content and low calorific value.
characteristic/ characteristics of Indian coal? The average ash content is approx. 40%. Indian coal has low sulphur
(2013) content and low moisture content.
1. High ash content. Majority of Indian coal have high fusion temperature.
2. Low sulphur content.

JOIN ME AT TELEGRAM – UPSC PRELIMS WITH NEELESH (AIR 442 UPSC CSE 2021)
https://t.me/UPSCPrelimsWithNeelesh
VISIT OUR WEBSITE – neeleshair442.com ©Copyright reserved with author
3. Low ash fusion temperature.
Select the correct answer using the codes
given below:
a. 1 and 2 only
b. 2 only
c. 1 and 3 only
d. 1,2 and 3

3 Consider the following statements: (2018) d As per the new Mining law – Mines and Minerals (Development and
1. In India, State Governments do not Regulation ) Amendment Act, 2015 -that came into effect from January
have the power to auction non-coal mines. 2015, the non-coal mines have to be auctioned by the respective state
2. Andhra Pradesh and Jharkhand do not Governments. Hence, statement 1 is not correct.
have gold mines. As per India Mineral Year Book 2020, by states, largest resources in terms
3. Rajasthan has iron ore mines. of gold ore (primary) are located in Bihar (44%) followed by Rajasthan
Which of the statements given above is/are (25%), Karnataka (21%), West Bengal & Andhra Pradesh (3% each) and
correct? Jharkhand (2%). The remaining 2% resources of ore are located in
a. 1 and 2 Chhattisgarh, Madhya Pradesh, Kerala, Maharashtra and Tamil Nadu.
b. 2 only
c. 1 and 3 Haematite and magnetite are the most important iron ores in India. About
d. 3 only 79% haematite ore deposits are found in the Eastern Sector (Assam,
Bihar, Chhattisgarh, Jharkhand, Odisha & Uttar Pradesh) while about 93%
magnetite ore deposits occur in Southern Sector (Andhra Pradesh, Goa,
Karnataka, Kerala & Tamil Nadu). Karnataka alone contributes 72% of
magnetite deposit in India. Of these, haematite is considered to be
superior because of its higher grade. Indian deposits of haematite belong
to the Precambrian Iron Ore Series and the ore is within banded iron ore
formations occurring as massive, laminated, friable and also in powdery
form.

Major reserves/resources of haematite are located in Odisha (7,559


million tonnes or 34%), Jharkhand (5,286 million tonnes or 23%),

JOIN ME AT TELEGRAM – UPSC PRELIMS WITH NEELESH (AIR 442 UPSC CSE 2021)
https://t.me/UPSCPrelimsWithNeelesh
VISIT OUR WEBSITE – neeleshair442.com ©Copyright reserved with author
Chhattisgarh (4,858 million tonnes or 22%), Karnataka (2,467 million
tonnes or 11%) and Goa (1,189 million tonnes or 5%). The balance 5%
resources of haematite are spread in Andhra Pradesh, Assam, Bihar,
Madhya Pradesh, Maharashtra, Meghalaya, Rajasthan, Telangana and
Uttar Pradesh.

India's 98% magnetite reserves/resources are located in five States,


namely, Karnataka (7,802 million tonnes or 72%) followed by Andhra
Pradesh (1,392 million tonnes or 13%), Rajasthan (617 million tonnes or
6%), Tamil Nadu (507 million tonnes or 5%) and Goa (266 million tonnes
or 2%). Assam, Bihar, Chhattisgarh, Jharkhand, Kerala, Maharashtra,
Meghalaya, Nagaland, Odisha and Telangana together account for the
remaining 2% resources

4 With reference to India, consider the following b Monazite is a brown crystalline mineral consisting of cerium, lanthanum,
statements? (2022) other rare earth elements, and thorium. Monazite usually occurs in small,
1. Monazite is source of rare earths. isolated crystals or grains that are resistant to weathering and become
2. Monazite contains thorium. concentrated in soils.
3. Monazite occurs naturally in the entire
Indian coastal sands in India. Statement 1 and 2 are correct.
4. In India, Government bodies only can Monazite is a major source of rare earths and thorium.
process or export monazite.
Which of the statements given above are It yields a number of rare-earth elements, such as neodymium and
correct? praseodymium. Both of these are in demand internationally for making
a. 1, 2 and 3 only high-performance rare-earth magnets (components of power wind
b. 1, 2 and 4 only turbines, electric vehicles and robotics)
c. 3 and 4 only Statement 3 is incorrect. Though monazite is found in most coastal areas
d. 1, 2, 3 and 4 of India. It is questionable to be found along entire coast.
The main mines are found along the coasts of southern India in Kerala,
Tamil Nadu and in Orissa.

JOIN ME AT TELEGRAM – UPSC PRELIMS WITH NEELESH (AIR 442 UPSC CSE 2021)
https://t.me/UPSCPrelimsWithNeelesh
VISIT OUR WEBSITE – neeleshair442.com ©Copyright reserved with author
Statement 4 is correct. Export of monazite without a license from AERB
is a violation of the Atomic Energy (Radiation Protection) Rules 2004.
Indian Rare Earths Limited (IREL), a wholly owned Public Sector
Undertaking of the Government of India (GOI) under DAE, is the only
entity which has been permitted to produce and process monazite, and
handle it for domestic use as well as for export.
5 Which one among the following industries is d
the maximum consumer of water in India?
(2013)
a. Engineering
b. Paper and pulp
c. Textiles
d. Thermal Power

6 Consider the following statements: (2013) b Gondwana beds are known for the coal deposits and not for natural gas.
1. Natural gas occurs in the Gondwana Kodarma has rich deposits of mica. Mica occurs in abundance in Kodarma.
beds. There are no signs of the petroleum in Dharwad region.
2. Mica occurs in abundance in Kodarma.
3. Dharwar is famous for petroleum.
Which of the statements given above is/are
correct?
JOIN ME AT TELEGRAM – UPSC PRELIMS WITH NEELESH (AIR 442 UPSC CSE 2021)
https://t.me/UPSCPrelimsWithNeelesh
VISIT OUR WEBSITE – neeleshair442.com ©Copyright reserved with author
a. 1 and 2
b. 2 only
c. 2 and 3
d. None

7 In which of the following regions of India are d India has several Shale Formations which seem to hold shale gas. The
shale gas resources found? (2016) Shale Gas Formations are spread over several sedimentary basins, such
1. Cambay Basin as, Gangetic plain, Gujarat, Rajasthan, Andhra Pradesh and other coastal
2. Cauvery Basin areas in the country including hydrocarbon-bearing ones - Cambay,
3. Krishna-Godavari Basin Assam Arkan & Damodar Basins have large shale deposits. Various
Select the correct answer using the code given developmental activities are going on in Gandhar area of Cambay Basin,
below: KG Basin, Cauvery Basin and Assam & Assam Arkan Basin.
a. 1 and 2 only
b. 3 only
c. 2 and 3 only
d. 1,2 and 3

INDIA: MANUFACTURING INDUSTRIES


Class XII: India People and Economy Chapter 8 (Manufacturing Industries)
Class X Chapter 6 (Manufacturing Industries)
Total Number of Questions Asked from 2011-22 1
My Instructions related to these Chapters There is very limited importance of it. Amongst the industries, focus
on Iron and Steel Industry, Cotton Textile, Sugar Industry. Read about
these in detail along with location factors etc.
PREVIOUS YEAR QUESTIONS RELATED TO THESE CHAPTERS OR TOPICS
S.No. Question Answer Explanation
1 In India, the steel production industry requires the import c Coking coal is an essential raw material for the making
of: (2015) of steel and Indian steel makers do not get them
a. Saltpetre adequately from domestic sources. Hence, they have to

JOIN ME AT TELEGRAM – UPSC PRELIMS WITH NEELESH (AIR 442 UPSC CSE 2021)
https://t.me/UPSCPrelimsWithNeelesh
VISIT OUR WEBSITE – neeleshair442.com ©Copyright reserved with author
b. Rock phosphate import them from abroad. Indian steel makers mostly
c. Coking coal use imported coking coal for use in the blast furnace.
d. All of the above
Production of one tonne of steel requires 0.8 tonne of
coking coal. India sources almost its entire requirement
of coking coal through imports. The limited reserves of
coking coal in the country and the poor quality (high ash
content and the poor coke strength) of the coal being
produced here make imports imperative.

INDIA: SERVICE SECTOR (TRANSPORT AND COMMUNICATION AND INTERNATIONAL TRADE ETC)
CLASS XII: India People and Chapter 10 (Transport and
Economy Communication
CLASS X Chapter 7 (Lifelines of National
Economy)
Total Number of Questions Asked from 2011-22 1
My Instructions related to these Chapters These chapters has limited importance. However, you must learn
about Golden Quadrilateral, North south and east-west corridor,
major National Highway, Railway zone and their headquarter,
national waterways of India.

PREVIOUS YEAR QUESTIONS RELATED TO THESE CHAPTERS OR TOPICS


S.No. Question Answer Explanation
1 Consider the following pairs: (2014) d NH 4 links four major Indian cities. They are Mumbai,
National Highway Cities Connected Pune, Bangalore and Chennai.
1. NH 4 : Chennai and Hyderabad NH 6 passes through the cities of Surat, Dhule, Amravati,
2. NH 6 : Mumbai and Kolkata Nagpur, Bhandara, Durg, Raipur, Kolkata.
3. NH 15 : Ahmedabad and Jodhpur NH 15 connects Samakhiali in Gujarat with Pathankot in
Which of the above pairs is/are correctly matched? Punjab.
a. 1 and 2 only
JOIN ME AT TELEGRAM – UPSC PRELIMS WITH NEELESH (AIR 442 UPSC CSE 2021)
https://t.me/UPSCPrelimsWithNeelesh
VISIT OUR WEBSITE – neeleshair442.com ©Copyright reserved with author
b. 3 only
c. 1,2 and 3
d. None

INDIA: PLANNING AND SUSTAINABLE DEVELOPMENT IN INDIAN CONTEXT AND GEOGRAPHICAL PERSPECTIVES

Class XII: India People and Economy Chapter 9 (Planning and


Sustainable Development in Indian
Context) + Chapter 12
(Geographical Perspective))
Total Number of Questions Asked from 2011-22 0
My Instructions related to these Chapters Not an important chapter. Still read the sustainable development
topic from it as it is always in news.

INDIA MISCELLANEOUS (NOT FROM NCERT THOUGH)


Total Number of Questions Asked from 2011-22 3
My Instructions related to these Chapters Tribe portion is not properly covered in NCERT. You may refer more
on tribes from UPSC and State PSC Booster Book topic Human
Geography (Page 132-135) for additional information. You can get its
free pdf from neeleshair442.com

JOIN ME AT TELEGRAM – UPSC PRELIMS WITH NEELESH (AIR 442 UPSC CSE 2021)
https://t.me/UPSCPrelimsWithNeelesh
VISIT OUR WEBSITE – neeleshair442.com ©Copyright reserved with author
PREVIOUS YEAR QUESTIONS RELATED TO THESE CHAPTERS OR TOPICS
S.No. Question Answer Explanation
1 Consider the following pairs: (2013) a - The Limbu (exonym) or Yakthung (endonym) are a
Tribe State Sino-Tibetan indigenous tribe (Bhot-Burmeli) of
1. Limboo (limbu) : Sikkim the Himalayan region of eastern Nepal, Sikkim, and
2. Karbi : Himachal Pradesh western Bhutan
3. Dongaria Kondh : Odisha - The Karbis or Mikir are one of the major ethnic
4. Bonda : Tamil Nadu communities in Northeast India, mostly concentrated in
Which of the above pairs are correctly matched? the hill district of Karbi Anglong of Assam.
a. 1 and 3 only - The Dangaria Kondh people are members of
b. 2 and 4 only the Kondhs. They are located in the Niyamgiri hills in the
c. 1,3 and 4 only state of Odisha (formerly Orissa) in India.
d. 1,2,3 and 4 - The Bonda (also known as the Bondo, Bondo
Poraja, Bhonda, or Remo) are a Munda ethnic group
approximately 12,000 (2011 census) who live in the
isolated hill regions of the Malkangiri district of
southwestern Odisha, India, near the junction of the
three states of Odisha, Chhattisgarh, and Andhra Pradesh.

2 Every year, a month long ecologically important b Every monsoon, the Gond and Korku tribes of Madhya
campaign/ festival is held during which certain Pradesh’s Betul and Harda districts celebrate Hari Jiroti.
communities/ tribes plant sapling of fruit bearing trees. It is a month- long festival of greenery, during which the
Which of the following are such communities/ tribes? tribals plant saplings of fruit- bearing trees.
(2014)
a. Bhutia and Lepcha
b. Gond and Korku
c. Irula and Toda
d. Sahariya and Agariya

JOIN ME AT TELEGRAM – UPSC PRELIMS WITH NEELESH (AIR 442 UPSC CSE 2021)
https://t.me/UPSCPrelimsWithNeelesh
VISIT OUR WEBSITE – neeleshair442.com ©Copyright reserved with author
3 With reference to the ‘Changpa’ community of India, b The Changpa are semi-nomadic Tibetan people. They
consider the following statements: (2014) are mainly found in the Changtang, a high plateau that
1. They live mainly in the State of Uttarakhand. stretches across the cold desert of Ladakh and also in
2. They rear the Pashmina goats that yield a fine some parts of Jammu and Kashmir. People from this
wool. tribe lived in the western regions of the autonomous
3. They are kept in the category of Scheduled Tribes. Tibetan region but were then forced by China to move
Which of the statements given above is/ are correct? out.
a. 1 only The military tensions between India and China caused
b. 2 and 3 only major disruptions in the life of tribal people of Changpa
c. 3 only whose major occupation is rearing of Pashmina goats
d. 1,2 and 3 which are found on high altitudes.
Tribal Ministry website lists Changpa under ST category
from Jammu and Kashmir.

JOIN ME AT TELEGRAM – UPSC PRELIMS WITH NEELESH (AIR 442 UPSC CSE 2021)
https://t.me/UPSCPrelimsWithNeelesh
VISIT OUR WEBSITE – neeleshair442.com ©Copyright reserved with author
WORLD GEOGRAPHY
WORLD: GEOGRAPHY AS A DISCIPLINE
CLASS XI(FUNDAMENTAL OF Chapter 1 (Geography as a Discipline)
PHYSICAL GEOGRAPHY)
Total Number of Questions Asked from 2011-22 0
My Instructions related to these Chapters This chapter is not important. You may ignore it too.
WORLD: THE EARTH
CLASS XI (FUNDAMENTAL OF Chapter 2 (The Origin and Evolution
PHYSICAL GEOGRAPH): of Earth) + Chapter 3 (Interior of the
Earth) + Chapter 4 (Distribution of
Oceans and Continents)
Class VII Chapter 2 (Inside Our Earth)
Class VI Chapter 1 (The Earth in the Solar
System) + Chapter 2 (Globe: Latitude
and Longitude) + Chapter 3 (Motion
of the Earth) + Chapter 4 (Maps) +
Chapter 5 (Major Domains of the
Earth)
Total Number of Questions Asked from 2011-22 6
My Instructions related to these Chapters This portion is very important. UPSC tend to ask simple, yet
conceptual questions from it. Since, limited type of questions are
framed from it, you must not make mistakes in understanding the
concepts. 4/6 questions can be deciphered from Class 6 th book. So,

JOIN ME AT TELEGRAM – UPSC PRELIMS WITH NEELESH (AIR 442 UPSC CSE 2021)
https://t.me/UPSCPrelimsWithNeelesh
VISIT OUR WEBSITE – neeleshair442.com ©Copyright reserved with author
you must not ignore these chapters. Supplement your knowledge
with the UPSC and State PSC Booster book by reading it selectively.

PREVIOUS YEAR QUESTIONS RELATED TO THESE CHAPTERS OR TOPICS


S.No. Question Answer Explanation
1 A person stood alone in a desert on a dark night and c You always see a pole star in the North direction.
wanted to reach his village which was situated 5 km east Keeping this in mind, you can solve this question easily
of the point where he was standing. He had no by applying logical reasoning.
instruments to find the direction but he located the
polestar. The most convenient way now to reach his village
is to walk in the: (2012)
(a) Direction facing the polestar
(b) Direction opposite to the polestar.
(c) Direction keeping the polestar to his left.
(d) Direction keeping the polestar to his right.

2 Variations in the length of daytime and nighttime from d There is a variation in the length of daytime and
season to season are due to: (2013) nighttime from season-to-season since the earth is
(a) Earth’s rotation on its axis. tilted at 23.5° on its axis, as it orbits the Sun. Different
(b) The earth’s revolution round the sun in an elliptical parts of the earth are nearer to the Sun, making it seem
manner. higher in the sky or lower in the sky. The closer you are
(c) Latitudinal position of the place. to the equator, the less of a difference there is in the
(d) Revolution of the earth on a tilted axis. length of the day throughout the year.

3 On 21st June, the Sun: (2019) a On the summer solstice, the Earth’s North Pole is at it’s
(a) Does not set below the horizon at the Arctic Circle. maximum tilt towards the Sun. At this time, The Sun
(b) Does not set below the horizon at Antarctic Circle. appears directly overhead at 23.5° north latitude, along
(c) Shines vertically overhead at noon on the Equator. the Tropic of Cancer. Around the Arctic circle, the
(d) Shines vertically overhead at the Tropic of location of sunrise and sunset start to converge in the
Capricorn. northern sky, until eventually the Sun never sinks below

JOIN ME AT TELEGRAM – UPSC PRELIMS WITH NEELESH (AIR 442 UPSC CSE 2021)
https://t.me/UPSCPrelimsWithNeelesh
VISIT OUR WEBSITE – neeleshair442.com ©Copyright reserved with author
the horizon. Hence, the appearance of the midnight sun
appears during this time.
4 In the northern hemisphere, the longest day of the year b 21st June is the longest day of the year in the Northern
normally occurs in the: (2022) Hemisphere, that is second half of the month of June.
(a) First half of the month of June More precisely, it is called the summer solstice.
(b) Second half of the month of June
(c) First half of the month of July
(d) Second half of the month of July

5 Which of the following phenomenon might have c As the continents drifted apart, it slowly took the
influenced the evolution of organisms? (2014) animals with them. Animals became isolated on the
1. Continental Drift large masses of land without control of the climate
2. Glacial Cycles change. Some continents went to cold areas, some to
Select the correct answer using the code given below: dry areas and some to tropical areas. As time went by,
(a) 1 only the animals learned to adapt to the different
(b) 2 only environments and they multiplied and bred according to
(c) Both 1 and 2 what animals survived best in each different type of
(d) Neither 1 nor 2 habitat. Also, because the continents were so far apart,
interbreeding became impossible so new species began
to appear in each isolated area. This was because both
random mutation and natural selection were contained
in a smaller space and continued without the genetic
influence of other species, which would have caused
much less variety. As time went on, new species
continued to appear and they became more and more
specialized for each new environment.

A glacial period (alternatively glacial or glaciation) is an


interval of time (thousands of years) within an ice
age that is marked by colder temperatures
and glacier advances. Interglacials, on the other hand, are

JOIN ME AT TELEGRAM – UPSC PRELIMS WITH NEELESH (AIR 442 UPSC CSE 2021)
https://t.me/UPSCPrelimsWithNeelesh
VISIT OUR WEBSITE – neeleshair442.com ©Copyright reserved with author
periods of warmer climate between glacial periods.
The Last Glacial Period ended about 15,000 years ago.
The Holocene is the current interglacial. A time with no
glaciers on Earth is considered a greenhouse climate state.
It has significantly affected the Evolution of Organism.

6 Consider the following statements: (2018) c - Geomagnetic reversal is the reversal of magnetic
1. The earth’s magnetic field has reversed every few north and the magnetic south, which happens in every
hundred thousand years. few hundred thousand years. Here the geographic north
2. When the earth was created more than 4000 and geographic south remains the same. Hence,
million years ago, there was 54% oxygen and no carbon statement 1 is correct.
dioxide. - The early atmosphere as hydrogen and helium
3. When living organisms originated, they modified and not Oxygen. When the gases and water vapour were
the early atmosphere of the earth. released during the cooling of the earth, it contained
Which of the statements given above is/are correct? water vapour, nitrogen, carbon dioxide, methane,
(a) 1 only ammonia and very little of free hydrogen. Thus,
(b) 2 and 3 only statement 2 is incorrect.
(c) 1 and 3 only - Respiration and microbial action by living
(d) 1, 2 and 3 organisms modified the early atmosphere of the earth.
Hence, statement 3 is correct.

WORLD: LANDFORMS
CLASS XI (FUNDAMENTAL OF Chapter 5 (Minerals and Rocks) +
PHYSICAL GEOGRAPHY Chapter 6 (Geomorphic Process) +
Chapter 7 (Landforms and their
Evolution)
CLASS VII Chapter 3 (Our Changing Earth)
CLASS VI Chapter 6 (Major Landforms of the
Earth)
JOIN ME AT TELEGRAM – UPSC PRELIMS WITH NEELESH (AIR 442 UPSC CSE 2021)
https://t.me/UPSCPrelimsWithNeelesh
VISIT OUR WEBSITE – neeleshair442.com ©Copyright reserved with author
Total Number of Questions Asked from 2011-22 1
My Instructions related to these Chapters As you can see, UPSC tend not to form typical questions from these
chapters. However, you must know – the major elements of the
earth crust, difference between igneous, sedimentary and
metamorphic rocks, dynamic processes involved in bringing
changes on earth, diastrophism, types of landform formed due to
erosional and depositional process, examples given in class VI of the
mountains etc. Do not go in depth. Instead focus on developing an
understanding. You will be able to solve the questions easily from
these chapters.

PREVIOUS YEAR QUESTIONS RELATED TO THESE CHAPTERS OR TOPICS


S.No. Question Answer Explanation
1 Consider the following: (2013) d Earth is subject to changes by electromagnetic radiation
1. Electromagnetic radiation. in the form of solar energy (exogenic factors) that causes
2. Geothermal energy. precipitation which leads to erosion and several other
3. Gravitational force. activities bringing dynamic changes.
4. Plate movements.
5. Rotation of the earth. Geothermal energy comes from the heat within the
6. Revolution of the Earth. earth. Some visible features of Geothermal Energy are
Which of the above are responsible for bringing dynamic volcanoes, hot springs, geysers, etc.
changes on the surface of the earth?
(a) 1, 2, 3 and 4 only Tides that cause the coastal erosion, occurs due to the
(b) 1, 3, 5 and 6 only gravitational pull of the moon.
(c) 2, 4, 5 and 6 only
(d) 1, 2, 3, 4, 5 and 6 Plate tectonics tells us that the earth’s rigid outer shell
(lithosphere) is broken into a mosaic of oceanic and
continental plates which can slide over the
aesthenosphere, which is the upper most layer of the
mantle. The plates are in constant motion. Where they

JOIN ME AT TELEGRAM – UPSC PRELIMS WITH NEELESH (AIR 442 UPSC CSE 2021)
https://t.me/UPSCPrelimsWithNeelesh
VISIT OUR WEBSITE – neeleshair442.com ©Copyright reserved with author
interact, along their margins, important geological
processes take place, such as the formation of mountain
belts, earthquakes and volcanoes. Earthquakes bring
dynamic changes in the earth.

Rotation of earth causes Coriolis Force that is


responsible for many things.
Revolution of earth is there to make changes in seasons
due to because of the axial tilt of the Earth.

All of the factors bring changes on earth in one way or


the other.

WORLD: CLIMATE
CLASS XI(FUNDAMENTAL OF Chapter 8 (Composition and
PHYSICAL GEOGRAPHY structure of Atmosphere) + Chapter
9 (Solar Radiation, Heat Balance
and Temperature + Chapter 10
(Atmospheric Circulation and
Weather Systems) + Chapter 11
(Water in the Atmosphere) +
Chapter 12 (World Climate and
Climate Change))
CLASS VII Chapter 4 (AIR)
Total Number of Questions Asked from 2011-22 13
My Instructions related to these Chapters This is one of the most important sections. A favorite of UPSC. You
must read these chapters in detail. Everything is important in these
chapters. UPSC tend to make questions from the lines given in these
JOIN ME AT TELEGRAM – UPSC PRELIMS WITH NEELESH (AIR 442 UPSC CSE 2021)
https://t.me/UPSCPrelimsWithNeelesh
VISIT OUR WEBSITE – neeleshair442.com ©Copyright reserved with author
chapters. While reading these chapters, you are advised to keep
revising the PYQ so that you read it in a right direction.

PREVIOUS YEAR QUESTIONS RELATED TO THESE CHAPTERS OR TOPICS


S.No. Question Answer Explanation
1 The jet aircrafts fly very easily and smoothly in the lower c The stratosphere is very dry; air which is present there
stratosphere. What could be the appropriate explanation? contains little water vapor. Because of this, few clouds are
(2011) found in this layer; almost all clouds occur in the lower, more
1. There are no clouds or water-vapour in the lower humid troposphere. The troposphere is an unstable layer
stratosphere. where the air is constantly moving. As a result, aircraft flying
2. There are no vertical winds in the lower through the troposphere may have a very bumpy ride, known
stratosphere. as turbulence. Because of this turbulence, most jet airlines fly
Which of the statements given above is/are correct in this higher above the Earth in the stratosphere. Here the air is
context? more still and clear as they can fly above the clouds.
(a) 1 only The increase in the temperature with height in the
(b) 2 only stratosphere makes this region very stable place where the air
(c) Both 1 and 2 tends not to overturn vertically. Thus, vertical winds are
(d) Neither 1 nor 2 almost absent in the Stratosphere.

2 Normally, the temperature decreases with the increase in c There are two reasons. First, the atmosphere is mainly heated
height from the earth’s surface, because: (2012) from the ground, rather than by the sun directly. Second, as
1. The atmosphere can be heated upwards only from the pressure of a gas decreases, so does its temperature.
the earth’s surface. Thus, the warmed air near the surface becomes less dense
2. There is more moisture in the upper atmosphere. and begins to rise, it expands and cools.
3. The air is less dense in the upper atmosphere. The thermosphere is a layer of the upper atmosphere that is
Select the correct answer using the codes given below: heated by direct solar radiation. However, it is at such a high
(a) 1 only altitude that it is effectively disconnected from the mixing and
(b) 2 and 3 only convection that occurs at lower levels. With altitude there is

JOIN ME AT TELEGRAM – UPSC PRELIMS WITH NEELESH (AIR 442 UPSC CSE 2021)
https://t.me/UPSCPrelimsWithNeelesh
VISIT OUR WEBSITE – neeleshair442.com ©Copyright reserved with author
(c) 1 and 3 only decrease in air pressure, so due to adiabatic expansion,
(d) 1,2 and 3 temperature declines.

3 Westerlies in southern hemisphere are stronger and a In the southern hemisphere, there is more ocean and less land
persistent than in northern hemisphere. Why? (2011) in comparison to the northern hemisphere. Due to this
1. Southern hemisphere has less landmass as reason, the Westerlies blow with much greater force in the
compared to northern hemisphere. southern hemisphere in comparison to the northern
2. Coriolis force is higher in southern hemisphere as hemisphere as there is less surface friction in the southern
compared to northern hemisphere. hemisphere. Higher the friction less the wind velocity.
Which of the statements given above is/are correct?
(a) 1 only In physics, the Coriolis force is an inertial or fictitious force
(b) 2 only that acts on objects in motion within a frame of reference that
(c) Both 1 and 2 rotates with respect to an inertial frame Deflection of an
(d) Neither 1 nor 2 object due to the Coriolis force is called the Coriolis effect.

The Coriolis effect characteristics can be summarized as


follows:

- Coriolis force is a fictitious force resulting from the


rotational movement of the earth.
- Coriolis effect is effective on objects that are in motion
such as wind, aircraft, ballistic and flying birds.
- Coriolis effect only affects the wind direction and not
the wind speed as it deflects the wind direction from the
expected path.
- The magnitude of Coriolis force is determined by wind
speed. The higher the wind speed, the greater the deflection.
- Coriolis effect is maximum at the poles and zero at the
equator.
- Coriolis force always acts in a direction that is
perpendicular to the moving object’s axis.

JOIN ME AT TELEGRAM – UPSC PRELIMS WITH NEELESH (AIR 442 UPSC CSE 2021)
https://t.me/UPSCPrelimsWithNeelesh
VISIT OUR WEBSITE – neeleshair442.com ©Copyright reserved with author
- Coriolis force is equal in both the hemispheres at their
respective latitudes.

4 In the South Atlantic and South-Eastern Pacific regions in b ITCZ stays at or near the equator and does not shift southward
tropical latitudes, cyclone does not originate. What is the over the south Atlantic or South Pacific region. Therefore,
reason? (2015) these regions do not have the ITCZ over them which is a major
(a) Sea surface temperatures are low. cause of the lack of cyclones.
(b) Inter-tropical convergence zone seldom occurs.
(c) Coriolis force is too weak
(d) Absence of land in those regions.

5 What explains the eastward flow of the equatorial counter b Convergence of the two equatorial currents leads to the
current? (2015) eastward flow of the equatorial counter currents.
(a) The Earth’s rotation on its axis.
(b) Convergence of the two equatorial currents.
(c) Difference in salinity of water.
(d) Occurrence of the belt of calm near the equator.

JOIN ME AT TELEGRAM – UPSC PRELIMS WITH NEELESH (AIR 442 UPSC CSE 2021)
https://t.me/UPSCPrelimsWithNeelesh
VISIT OUR WEBSITE – neeleshair442.com ©Copyright reserved with author
6 Consider the following statements: (2015) b Westerlies are the permanent winds that blow between the
1. The winds which blow between 30° N and 60° S 30° to 60° latitude in both the hemispheres, i.e Southern and
latitudes throughout the year are known as westerlies. Northern Hemisphere.
2. The moist air masses that cause winter rains in
North-Western region of India are part of westerlies.
Which of the statements given above is/are correct?
(a) 1 only
(b) 2 only
(c) Both 1 and 2
(d) Neither 1 nor 2

Statement 1 is saying 30° North to 60° South, which is


incorrect.
Western disturbance causes winter rainfall across northwest.
These disturbances originate over the Mediterranean Sea and
are bought into India by westerly jet stream. Western
disturbances are low-pressure areas embedded in the
westerlies, the planetary winds that blow from west to east
between 30°-60° latitude. Hence, 2 is correct.
7 Consider the following statements: (2021) c In tropical zone, the trade winds drive both North and South
1. In the tropical zone, the western sections of the equatorial currents westward, thus transporting warm ocean-
oceans are warmer than the eastern sections owing to the surface waters in the direction. So, in tropical zones, the
influence of trade winds. western section of ocean is warmer than the eastern section
2. In the temperate zone, westerlies make the due to trade winds. So, statement 1 Is correct.
eastern sections warmer than the western sections.
Which of the statements given above is/are correct? The westerlies play an important role in carrying the warm,
(a) 1 only equatorial waters and winds to the western coasts of

JOIN ME AT TELEGRAM – UPSC PRELIMS WITH NEELESH (AIR 442 UPSC CSE 2021)
https://t.me/UPSCPrelimsWithNeelesh
VISIT OUR WEBSITE – neeleshair442.com ©Copyright reserved with author
(b) 2 only continents, especially in the southern hemisphere because of
(c) Both 1 and 2 its vast oceanic expanse. Thus, in the temperate zone,
(d) Neither 1 nor 2 westerlies make the eastern sections of oceans warmer than
the western sections. So, statement 2 is also correct.

8 The annual range of temperature in the interior of the a The annual range of temperature in the interior of the
continents is high as compared to coastal areas. What continents is high as compared to coastal areas because land
is/are the reason/reasons? (2013) areas heat up and cool off more quickly than oceans.
1. Thermal difference between land and water. Therefore, in the winter, the interiors of continents will be
2. Variation in altitude between continents and much colder than along the coast. Similarly, in the summer,
oceans. the interiors heat up much more than the coastal areas. The
3. Presence of strong winds in the interior. colder winter temperatures and warmer summer
4. Heavy rains in the interior as compared to coasts. temperatures of the interior result in a greater range of
Select the correct answer using the codes given below: temperature.
(a) 1 only
(b) 1 and 2 only
(c) 2 and 3 only
(d) 1, 2, 3 and 4

9 Consider the following statements: (2020) c Jet streams are fast flowing, narrow meandering air currents
1. Jet streams occur in the Northern Hemisphere in the atmosphere of the Earth. It is a geostrophic wind
only. blowing horizontally through the upper layers of the
2. Only some cyclones develop an eye. troposphere. It is found in both, the northern and the
3. The temperature inside the eye of a cyclone is southern hemispheres, however the jet streams in the north
nearly 10°C lesser than that of the surroundings. are more forceful due to greater temperature gradients.
Which of the statements given above is/are correct?
(a) 1 only Generally mature storms have well developed eye, however
(b) 2 and 3 only extra-tropical cyclones may not always have an eye.
(c) 2 only
(d) 1 and 3 only The eye is the region of lowest surface pressure and warmest
temperatures aloft (in the upper levels)- the eye temperature

JOIN ME AT TELEGRAM – UPSC PRELIMS WITH NEELESH (AIR 442 UPSC CSE 2021)
https://t.me/UPSCPrelimsWithNeelesh
VISIT OUR WEBSITE – neeleshair442.com ©Copyright reserved with author
may be 10°C warmer or more at the higher altitude than the
surrounding environment but only 0-20C warmer at the
surface in the tropical cyclone.

10 What could be main reason/reasons for the formation of a The main reason for the formation of African and Eurasian
African and Eurasian desert belt? (2011) desert belt is that it is located in the subtropical high-pressure
1. It is located in the sub-tropical high-pressure cells. cells. This is zone of hot, dry air that forms as the warm air
2. It is under the influence of warm ocean currents. descending from the tropics becomes hotter. Because hot air
Which of the statements given above is/are correct in this can hold more water vapour, it is relatively dry. The heavy rain
context? along the equator also removes most of the excess moisture.
(a) 1 only The dominant winds in the Subtropical high are called
(b) 2 only westerlies.
(c) Both 1 and 2 African and Eurasian desert belts are under the influence of
(d) Neither 1 nor 2 cold ocean currents, which leads to higher aridity. So,
statement 2 is incorrect.

11 Consider The following statements: (2022) d Low, thick clouds primarily reflect the solar radiation and cool
1. High clouds primarily reflect solar radiation and the surface of the earth.
cool the surface of the Earth. High clouds transmit the incoming solar radiation. And at the
2. Low clouds have a high absorption of infrared same time, they trap some of the outgoing infrared radiation
radiation emanating from the Earth’s surface and thus emitted by the earth and radiate it back downward, thereby
cause warming effect. warming the surface.
Which of the statements given above is/are correct?
(a) 1 only
(b) 2 only
(c) Both 1 and 2
(d) Neither 1 nor 2

12 Why are dewdrops not formed on a cloudy night? (2019) b On a cloudy weather condition, terrestrial radiation is
(a) Clouds absorb the radiation released from the radiated back to the earth’s surface after reflection from
Earth’s surface. clouds. This leads to formation of hothouse condition due to

JOIN ME AT TELEGRAM – UPSC PRELIMS WITH NEELESH (AIR 442 UPSC CSE 2021)
https://t.me/UPSCPrelimsWithNeelesh
VISIT OUR WEBSITE – neeleshair442.com ©Copyright reserved with author
(b) Clouds reflect back the Earth’s radiation. which temperature on earth’s surface is relatively higher. All
(c) The Earth’s surface would have low temperature these conditions become unfavorable for the formation of
on cloudy nights. dew droplets on a cloudy night.
(d) Clouds deflect the blowing wind to ground level.

13 During thunderstorm, the thunder in the skies is produced d Thunder is the rumbling or crack of sound that can usually be
by the: (2013) heard from the sky during a storm. Thunder is caused because
1. Meeting of cumulonimbus clouds in the sky. lightening heats up the air, to about 30000°C, causing it to
2. Lightning that separates the nimbus clouds. expand quickly. The rumbling occurs as the sound passes
3. Violent upward movement of air and water through atmospheric layers at different temperatures.
particles.
Select the correct answer using the codes given below:
(a) 1 only
(b) 2 and 3
(c) 1 and 3
(d) None of the above produces thunder.

WORLD: WATER (OCEAN)


CLASS XI (FUNDAMENTAL OF Chapter 13 (Water (Oceans) +
PHYSICAL GEOGRAPHY Chapter 14 (Movements of the
Ocean Water))
CLASS VII Chapter 5 (Water)
Total Number of Questions Asked from 2011-22 6
My Instructions related to these Chapters This is also a favorite portion of UPSC. Keeping in Mind the PYQ, you can
easily have a directed/focussed preparation. You must prepare these
chapters in detail. Focus on distribution of water on earth’s surface,
factors affecting temperature distribution, horizontal and vertical
distribution of temperature, salinity of ocean water and its distribution,
waves, relation between gravitation and tides, ocean currents – forces
which influence them, major ocean current etc.

JOIN ME AT TELEGRAM – UPSC PRELIMS WITH NEELESH (AIR 442 UPSC CSE 2021)
https://t.me/UPSCPrelimsWithNeelesh
VISIT OUR WEBSITE – neeleshair442.com ©Copyright reserved with author
PREVIOUS YEAR QUESTIONS RELATED TO THESE CHAPTERS OR TOPICS
S.No. Question Answer Explanation
1 Consider the following factors: (2012) b Ocean currents are influenced by two types of forces namely:
1. Rotation of the Earth. Primary forces:
2. Air pressure and wind. - Heating by solar energy: Heating by solar energy causes the
3. Density of ocean water. water to expand. That is why, near the equator the ocean water is about
4. Revolution of the Earth. 8 cm higher in level than in the middle latitudes. This causes a very slight
Which of the above factors influence the ocean gradient and water tends to flow down the slope.
currents? - Wind: Wind blowing on the surface of the ocean pushes the
(a) 1 and 2 only water to move. Friction between the wind and the water surface affects
(b) 1,2 and 3 the movement of the water body in its course.
- Gravity: Gravity tends to pull the water down the pile and create
(c) 1 and 4 gradient variation.
(d) 2,3 and 4 - Coriolis force: The Coriolis force intervenes and causes the
water to move to the right in the northern hemisphere and to the left
in the southern hemisphere.
Secondary forces:
- Differences in water density: It affects vertical mobility of ocean
currents.
Water with high salinity is denser than water with low salinity and in the
same way cold water is denser than warm water.
Denser water tends to sink, while relatively lighter water tends to rise.
- Temperature of water: Cold-water Ocean currents occur when
the cold water at the poles sinks and slowly moves towards the equator.
Warm-water currents travel out from the equator along the surface,
flowing towards the poles to replace the sinking cold water

2 With reference to Ocean Mean Temperature b OMT is measured upto a depth of 26°C isotherm, is more stable and
(OMT), which of the following statements is/are consistent, and the spatial spread is also less. The 26°C isotherm is seen
correct? (2020) at depths varying from 50-100 meters. During January-March, the mean

JOIN ME AT TELEGRAM – UPSC PRELIMS WITH NEELESH (AIR 442 UPSC CSE 2021)
https://t.me/UPSCPrelimsWithNeelesh
VISIT OUR WEBSITE – neeleshair442.com ©Copyright reserved with author
1. OMT is measured up to a depth of 26°C 26°C isotherm depth in the South-western Indian Ocean is 59 meters.
isotherm which is 129 meters in the south- Hence, statement 1 is incorrect.
western Indian Ocean during January- March.
2. OMT collected during January-March can OMT has better ability to predict monsoon than the sea surface
be used in assessing whether the amount of temperature (SST). SST has 60% success rate of predicting the monsoon,
rainfall in monsoon will be less or more than a while OMT has 80% success rate. This is because OMT measures the
certain long-term mean. ocean’s thermal energy whereas SST is influenced by surface winds,
Select the correct answer using the code given evaporation, thick clouds, etc.
below:
(a) 1 only
(b) 2 only
(c) Both 1 and 2
(d) Neither 1 nor 2

3 The most important fishing grounds of the world c When warm and cold currents meet, dense fog is formed. Reduced
are found in the regions where: (2013) visibility avoid the ships to enter in that region. On the other hand, such
(a) Warm and cold atmospheric currents areas are ideal for fishing. A fishing bank is shallow sea area which
meet. abounds in fish. Cold currents bring abundant fish food called plankton
(b) Rivers drain out large amounts of from the cold polar areas and this supports large fish populations.
freshwater into the sea.
(c) Warm and cold oceanic currents meet.
(d) Continental shelf is undulating.

4 Tides occur in the oceans and seas due to which d All the given forces affect the formation of the tides. It can be easily
among the following? (2015) solved by covering your NCERT chapters with understanding.
1. Gravitational force of the Sun.
2. Gravitational force of the Moon.
3. Centrifugal force of the Earth.
Select the correct answer using the code given
below:
(a) 1 only

JOIN ME AT TELEGRAM – UPSC PRELIMS WITH NEELESH (AIR 442 UPSC CSE 2021)
https://t.me/UPSCPrelimsWithNeelesh
VISIT OUR WEBSITE – neeleshair442.com ©Copyright reserved with author
CIVIL SERVICES WITH NEELESH
(TELEGRAM CHANNEL – UPSC PRELIMS WITH NEELESH)
(YOUTUBE CHANNEL – CIVIL SERVICES WITH NEELESH)
Note – All our initiatives (both free and paid) are on these channels only.

OUR INITIATIVES (Running Currently)

1. PRELIMS MENTORSHIP 2024 (BOTH FREE AND PAID)


2. ETHICS AND ESSAY EVALUATION – ER NEELESH AIR 442 UPSC_CSE2021 (https://t.me/UpscWithNeelesh_AIR442)
3. SOCIOLOGY FRAMEWORK - TELEGRAM – SOCIOLOGY WITH NEELESH (https://t.me/SociologyWithNeelesh)
4. FREE CSAT PROGRAM – ON YOUTUBE CHANNEL – CIVIL SERVICES WITH NEELESH
5. INTEGRATED MENTORSHIP FOR 2025 (PRELIMS AND MAINS) – YEARLONG MENTORSHIP
6. SHORT NOTES FOR ALL SUBJECTS (HANDWRITTEN)

UPCOMING INITIATIVES

1.PRELIMS TEST SERIES FOR 2025


2. MAINS TEST SERIES FOR 2025
3. ESSAY VIDEO BATCH FOR 2024 AND 2025
4. ETHICS VIDEO BATCH FOR 2024 AND 2025
5. SOCIOLOGY VIDEO BATCH FOR 2025 BASED ON PYQ
6. CSAT VIDEO BATCH
7. MAINS MARATHON 2024 AND MUCH MORE
(b) 2 and 3 only
(c) 1 and 3 only
(d) 1, 2 and 3

5 On the planet earth, most of the freshwater c Out of all the water on Earth, water in the ocean is 97.25%, in ice caps
exists as ice caps and glaciers. Out of the and glaciers 2.05%, in ground water 0.68%, in lakes 0.01%, in soil
remaining freshwater, the largest proportion: moisture 0.005%, in atmosphere 0.001%, in steams and rivers 0.0001%
(2013) and in biosphere 0.00004 percent.
(a) Is found in atmosphere as moisture and
clouds.
(b) Is found in freshwater lakes and rivers.
(c) Exists as groundwater.
(d) Exists as soil moisture.

6 With reference to the water on the planet Earth, b Refer to explanation above.
consider the following statements: (2021)
1. The amount of water in the rivers and
lakes is more than the amount of groundwater.
2. The amount of water in polar ice caps
and glaciers is more than the amount of
groundwater.
Which of the statements given above is/are
correct?
(a) 1 only
(b) 2 only
(c) Both 1 and 2
(d) Neither 1 nor 2

JOIN ME AT TELEGRAM – UPSC PRELIMS WITH NEELESH (AIR 442 UPSC CSE 2021)
https://t.me/UPSCPrelimsWithNeelesh
VISIT OUR WEBSITE – neeleshair442.com ©Copyright reserved with author
WORLD: LIFE ON THE EARTH
CLASS XI (FUNDAMENTAL OF Chapter 15 (Life on the Earth) +
PHYSICAL GEOGRAPHY Chapter 16 (Biodiversity and
Conservation)

CLASS VII Chapter 1 (Environment) + Chapter


6 (Natural Vegetation and Wildlife)
+ Chapter 8 (Human Environment
Interactions: The Tropical and the
Subtropical Region) + Chapter 9
(Life in the desert)
Total Number of Questions Asked from 2011-22 5
My Instructions related to these Chapters Vegetation is one of the favorite areas of UPSC. So, you must read
these chapters in detail. Know the climatic conditions and the main
flora and fauna found at these types of places.

PREVIOUS YEAR QUESTIONS RELATED TO THESE CHAPTERS OR TOPICS


S.No. Question Answer Explanation
1 Which one of the following is the characteristic climate of the d Tropical Savannah Region is characterized by definite
Tropical Savannah Region? (2012) dry and wet season.
(a) Rainfall throughout the year. Refer to your original source for the climatic condition
(b) Rainfall in winter only of various regions. It is important.
(c) An extremely short dry season.
(d) A definite dry and wet season.

2 The vegetation of savannah consists of grassland with c A savanna or savannah is a mixed woodland-grassland
scattered small trees, but extensive areas have no trees. The (i.e. Grassy woodland) ecosystem characterised by the
forest development in such areas is generally kept in check trees being sufficiently widely spaced so that the canopy
does not close. The open canopy allows sufficient light

JOIN ME AT TELEGRAM – UPSC PRELIMS WITH NEELESH (AIR 442 UPSC CSE 2021)
https://t.me/UPSCPrelimsWithNeelesh
VISIT OUR WEBSITE – neeleshair442.com ©Copyright reserved with author
by one or more or a combination of some conditions. Which to reach the ground to support an unbroken herbaceous
of the following are such conditions? (2021) layer consisting primarily of grasses.
1. Burrowing animals and termites. The threat to the savannah includes
2. Fire - Fire
3. Grazing herbivores - Grazing and browsing animals
4. Seasonal rainfall - Tree clearing
5. Soil properties - Exotic Plant Species
Select the correct answer using the code given below: - Climate change
(a) 1 and 2
(b) 4 and 5
(c) 2, 3 and 4
(d) 1, 3 and 5

3 Which of the following is/are unique characteristic/ d The equatorial forests are spread over just 6 % of the
characteristics of equatorial forests? (2013) Earth's surface. They are referred to as Rain Forest,
1. Presence of tall, closely set trees with crowns forming Selvas, or the Evergreen Forest and are treasures of
a continuous canopy. biodiversity. The equatorial area stretches out from 0°
2. Coexistence of a large number of species. to 10° in the North and South.
3. Presence of numerous varieties of epiphytes. The constantly high temperature and all year-round
Select the correct answer using the codes given below: rainfall helps to produce the most luxuriant evergreen
(a) 1 only vegetation in the region. These forests are known as
(b) 2 and 3 only equatorial evergreen forests. There are tall and closely
(c) 1 and 3 only set trees with crowns forming a canopy. There are
(d) 1, 2 and 3 number of species which co-exist together. Numerous
varieties of epiphytes are also present here.

4 “Each day is more or less the same, the morning is clear and b Equatorial regions have the climate that is hot and wet
bright with a sea breeze; as the Sun climbs high in the sky, all year round. One day in an equatorial climate can be
heat mounts up, dark clouds form, then rain comes with very similar to the next, while the change in
thunder and lightning. But rain is soon over.” (2015) temperature between day and night may be larger than
the average change in temperature along the year. By

JOIN ME AT TELEGRAM – UPSC PRELIMS WITH NEELESH (AIR 442 UPSC CSE 2021)
https://t.me/UPSCPrelimsWithNeelesh
VISIT OUR WEBSITE – neeleshair442.com ©Copyright reserved with author
Which of the following regions is described in the above noon dark clouds surround and bring heavy but short
passage? duration rainfall.
(a) Savannah
(b) Equatorial
(c) Monsson
(d) Mediterranean

5 “Climate is extreme, rainfall is scanty and the people used to b In the Siberian Tundra, rainfall is scanty and climate is
be nomadic herders.” (2013) extreme. But the people were not nomadic herders.
The above statements best describe which of the following Their primary occupation is hunting and gathering.
regions? Hence, option (d) is not correct.
(a) African Savannah In the North American Prairies, rainfall is scanty and
(b) Central Asian Steppe climate is extreme. The farmers here use mechanized
(c) North American Prairie equipment and do wheat cultivation on large scale.
(d) Siberian Tundra Hence, (c) is not correct.
All the conditions given above is correct for the Central
Asian Steppe region.

WORLD: HUMAN RESOURCES


CLASS XII: FUNDAMENTALS OF Chapter 1(Human Geography-
HUMAN GEOGRAPHY Nature and Scope) + Chapter 2 (The
World Population Distribution,
Density and Growth) + Chapter 3
(Population Composition) + Chapter
4 (Human Development) + Chapter
10 (Human Settlement)
CLASS VIII Chapter 6 (Human Resources)
Total Number of Questions Asked from 2011-22 0
My Instructions related to these Chapters This is same as the Indian portion of Geography containing the
Population chapters. UPSC has not asked questions from these

JOIN ME AT TELEGRAM – UPSC PRELIMS WITH NEELESH (AIR 442 UPSC CSE 2021)
https://t.me/UPSCPrelimsWithNeelesh
VISIT OUR WEBSITE – neeleshair442.com ©Copyright reserved with author
section in the last ten years. So, you may just have a brief reading
and move forward.

WORLD: RESOURCES
CLASS VIII (Chapter 1: Resources) + Chapter 2
(Land, Soil, Water, Natural
Vegetation and Wildlife Resources) +
Chapter 3 (Mineral and Power
Resources)
Total Number of Questions Asked from 2011-22 0
My Instructions related to these Chapters Simply understand the basics and move forward.
WORLD: PRIMARY ACTIVITIES
CLASS XII: FUNDAMENTALS OF Chapter 5 (Primary Activities)
HUMAN GEOGRAPHY
CLASS VIII Chapter 4 (Agriculture)
Total Number of Questions Asked from 2011-22 5
My Instructions related to these Chapters Agriculture remains the favorite of UPSC. So, read the chapter
properly.

PREVIOUS YEAR QUESTIONS RELATED TO THESE CHAPTERS OR TOPICS


S.No. Question Answer Explanation
1 With reference to micro-irrigation, which of the c Micro-irrigation is the slow application of water as discrete or
following statements is/are correct? (2011) continuous drips, tiny streams or miniature spray on the soil by
1. Fertilizer/ nutrient loss can be reduced. surface drip and micro-sprinkler systems. As the agriculture sector
2. It is the only means of irrigation in dry land consumes 80% of the freshwater in India, micro-irrigation is often
farming. promoted by central and state governments as a way to tackle the
3. In some areas of farming, receding of ground growing water crisis. This is because drip and sprinkler irrigation
water table can be checked.
JOIN ME AT TELEGRAM – UPSC PRELIMS WITH NEELESH (AIR 442 UPSC CSE 2021)
https://t.me/UPSCPrelimsWithNeelesh
VISIT OUR WEBSITE – neeleshair442.com ©Copyright reserved with author
Select the correct answer using the codes given deliver water to farms in far lesser quantities than conventional
below: gravity flow irrigation.
(a) 1 only
(b) 2 and 3 only Benefits of micro irrigation systems:
(c) 1 and 3 only - Water saving: Micro-irrigation (MI) is proved to be an
(d) 1, 2 and 3 efficient method in saving water and increasing water use
efficiency as compared to the conventional surface method of
irrigation, where water use efficiency is only about 35-40%.
- Increased irrigation efficiency: The on-farm irrigation
efficiency of properly designed and managed drip irrigation
systems is estimated to be about 90%. Farmers using a pumping
system to irrigate their fields should ensure that the pump and
pipe size are fitting with their needs, thus avoiding water and
energy overuse and consequent leakages.
- Higher yields: The yields are higher than traditional flood
irrigation. Productivity gains due to use of micro-irrigation is
estimated to be in the range of 20 to 90% for different crops. Yields
of crops increase up to 45% in wheat, 20% in gram and 40% in
soybean.
- Less water loss: There is also less loss of water due to
reduction lesser evaporation, run off, and by deep percolation.
- Energy efficient: The reduction in water consumption in
micro-irrigation also reduces the energy use (electricity) that is
required to lift water from irrigation wells.
- Lower consumption of fertilizers: An efficient drip
irrigation system reduces consumption of fertilizer through
fertigation.
- Weed and disease reduction: It helps in inhibiting growth
of weeds as it keeps limited wet areas. Under this condition the
incidence of disease is also reduced.

JOIN ME AT TELEGRAM – UPSC PRELIMS WITH NEELESH (AIR 442 UPSC CSE 2021)
https://t.me/UPSCPrelimsWithNeelesh
VISIT OUR WEBSITE – neeleshair442.com ©Copyright reserved with author
- Cost savings: There are substantial reductions in irrigation
costs and savings on electricity and fertilizers.
- Precision farming: Emerging computerized GPS-based
precision irrigation technologies for self-propelled sprinklers and
micro-irrigation systems will enable growers to apply water and
agrochemicals more precisely and site specifically to match soil
and plant status and needs as provided by wireless sensor
networks.

2 Salinization occurs when the irrigation water b Saline soils are also known as Usara soil or infertile soil as it does
accumulated in the soil evaporates, leaving behind not support any vegetation growth due to very high salt contents.
salts and minerals. What are the effects of salinization Saline soils formed in the following conditions:
on the irrigated land? (2011) - Excessive irrigation with dry climatic conditions promotes
(a) It greatly increases the crop production. capillary actions which lead to the accumulation of salt in the top
(b) It makes some soil impermeable. layer of soils.
(c) It raises the water table. - In poor drainage areas or waterlogging or swampy areas;
(d) It fills the air spaces in the soil with water. soluble salt in water gets accumulated in the soil that leading to a
change of alluvial soil or other soils into saline soils.
- Intensive cultivation with excess uses of irrigation, alluvial
soils becoming saline.
In the delta region or coastal region, soils get converted into saline
soil due to the intrusion of seawater into the soil.

The following are characteristics:


- Saline soil contains sodium, potassium, and magnesium in
a larger proportion. It is poor in nitrogen and calcium. Thus, soil
becomes infertile, and hence, do not support vegetation growth.
- Structure: sandy to loamy.
- Lack of nitrogen and calcium.
- To overcome problems, Gypsum is added to solve the
problems of salinity in soils.

JOIN ME AT TELEGRAM – UPSC PRELIMS WITH NEELESH (AIR 442 UPSC CSE 2021)
https://t.me/UPSCPrelimsWithNeelesh
VISIT OUR WEBSITE – neeleshair442.com ©Copyright reserved with author
3 With reference to agricultural soils, consider the b Plants obtain nutrients from two natural sources: organic matter
following statements: (2018) and minerals. Organic matter includes any plant or animal
1. A high content of organic matter in soil material that returns to the soil and goes through the
drastically reduces its water holding capacity. decomposition process. In addition to providing nutrients and
2. Soil does not play any role in the sulphur cycle. habitat to organisms living in the soil, organic matter also binds
3. Irrigation over a period of time can contribute soil particles into aggregates and improves the water holding
to the salinization of some agricultural lands. capacity of soil.
Which of the statements given above is/are correct?
(a) 1 and 2 only The sulphur cycle is the collection of processes by which sulphur
(b) 3 only moves to and from rock, waterways and living systems. The
(c) 1 and 3 only isotopic composition of sedimentary sulphides provides primary
(d) 1, 2 and 3 information on the evolution of the sulphur cycle.

Too much salt can reduce crop production and water infiltration
on soils that have been irrigated for 20 years or more. Too little
salt can also result in a chemically compacted soil. The level of
sodium (Na) in a soil can result in soil particles or even layers of
differing soil profiles forming a compacted layer that the roots do
not penetrate.

4 Which of the following is the chief characteristic of c A "mixed farming" system includes agricultural production,
‘mixed farming’? (2012) livestock, poultry, fisheries, beekeeping, and other activities on a
(a) Cultivation of both cash crops and food crops. single farm in order to support and meet the farmer's needs in as
(b) Cultivation of two or more crops in the same many ways as possible. The major objective of mixed farming is
field. subsistence. Higher profitability without compromising the
(c) Rearing of animals and cultivation of crops ecological balance is crucial for this farming system.
together.
(d) None of the above.

JOIN ME AT TELEGRAM – UPSC PRELIMS WITH NEELESH (AIR 442 UPSC CSE 2021)
https://t.me/UPSCPrelimsWithNeelesh
VISIT OUR WEBSITE – neeleshair442.com ©Copyright reserved with author
5 Which one of the following groups of plants was a "New World" means the Americas which were discovered by
domesticated in the ‘New World’ and introduced into European Powers such as Spain, Portuguese and French; Tobacco,
the ‘Old World’? (2019) Cocoa and Rubber were native crops of Americas and were
(a) Tobacco, cocoa and rubber introduced in the Old World (Asia).
(b) Tobacco, cotton and rubber
(c) Cotton, coffee and sugarcane
(d) Rubber, coffee and wheat

WORLD SECONDARY ACTIVITIES


CLASS XII: FUNDAMENTALS OF Chapter 6 (Secondary Activities)
HUMAN GEOGRAPHY
CLASS VIII Chapter 5 (Industries)
Total Number of Questions Asked from 2011-22 0
My Instructions related to these Chapters Just have a brief reading and move forward

WORLD TERTIARY AND OTHER SECTORS


CLASS XII: FUNDAMENTALS OF Chapter 7 (Tertiary and Quaternary
HUMAN GEOGRAPHY Activities) + Chapter 8 (Transport and
Communication) + Chapter 9
(International Trade)
CLASS VII Chapter 7 (Human Environment –
Settlement, Transport and
Communication)
Total Number of Questions Asked from 2011-22 0
My Instructions related to these Chapters Just have a brief reading and move forward

JOIN ME AT TELEGRAM – UPSC PRELIMS WITH NEELESH (AIR 442 UPSC CSE 2021)
https://t.me/UPSCPrelimsWithNeelesh
VISIT OUR WEBSITE – neeleshair442.com ©Copyright reserved with author
Map QUESTIONS (LOCATION BASED) (NOT PROPERLY GIVEN IN NCERT)
Total Number of Questions Asked from 2011-22 16
My Instructions related to these Chapters This portion has various Map based questions. There are still more
areas we can explore. You Should focus on important plateau,
mountains, highlands, deserts, places in news, countries around
important seas, important rivers etc.

PREVIOUS YEAR QUESTIONS RELATED TO THESE CHAPTERS OR TOPICS


1 Between India and East Asia, the navigation-time c The Strait of Malacca is a narrow stretch of water, 500 mi (800 km) long and
and distance can be greatly reduced by which of from 40 to 155 mi (65–250 km) wide, between the Malay Peninsula
the following? (2011) (Peninsular Malaysia) to the northeast and the Indonesian island of Sumatra
1. Deeping the Malacca straits between to the southwest, connecting the Andaman Sea (Indian Ocean) and the
Malaysia and Indonesia. South China Sea (Pacific Ocean).
2. Opening a new canal across the Kra As the main shipping channel between the Indian and Pacific oceans, it is
isthmus between the Gulf of Siam and Andaman one of the most important shipping lanes in the world.
Sea. Kra Canal also known as Thai Canal or Kra Isthmus Canal refers to a proposed
Which of the statements given above is/are canal project that aims to connect the Gulf of Thailand with the Andaman
correct? Sea across the Kra Isthmus in southern Thailand.
a. 1 only
b. 2 only
c. Both 1 and 2
d. Neither 1 nor 2

JOIN ME AT TELEGRAM – UPSC PRELIMS WITH NEELESH (AIR 442 UPSC CSE 2021)
https://t.me/UPSCPrelimsWithNeelesh
VISIT OUR WEBSITE – neeleshair442.com ©Copyright reserved with author
2 Which one of the following pairs is correctly b The Atlas Mountain is a mountain range across the northwestern stretch of
matched? (2013) Africa extending about 2500 km trough Morocco, Algeria and Tunisia.
Geographical Region Abyssinian Plateau : Ethiopia
feature Guiana Highlands : Omicco Basin
(a) Abyssinian Arabia Okavango Basin : South-west Africa
Plateau
(b) Atlas Mountains North-western Africa
(c) Guiana South-Western Africa
Highlands
(d) Okavango Basin Patagonia
3 Turkey is located between: (2014) b Turkey is bounded on the north by the Black Sea, on the northeast by
(a) Black Sea and Caspian Sea Georgia and Armenia, on the east by Azerbaijan and Iran, on the southeast
(b) Black Sea and Mediterranean Sea by Iraq and Syria, on the southwest and west by the Mediterranean Sea and
(c) Gulf of Suez and Mediterranean Sea the Aegean Sea, and on the northwest by Greece and Bulgaria.
(d) Gulf of Aqaba and Dead Sea You are advised to refer to your ATLAS.

JOIN ME AT TELEGRAM – UPSC PRELIMS WITH NEELESH (AIR 442 UPSC CSE 2021)
https://t.me/UPSCPrelimsWithNeelesh
VISIT OUR WEBSITE – neeleshair442.com ©Copyright reserved with author
4 They are known as ‘Golan Heights’ sometimes b The Golan Heights is a rocky plateau in south-western Syria, about 60km (40
appears in the news in the context of the events miles) south-west of Damascus and covers about 1,000 sq km. It has a
related to: (2015) political and strategic significance which belies its size. Israel seized the
(a) Central Asia
(b) Middle East
(c) South-East Asia
(d) Central Africa

Golan Heights from Syria in the closing stages of the 1967 Six-Day War. Most
of the Syrian Arab inhabitants fled the area during the conflict. It is in Middle
East.
5 What is the correct sequence of occurrence of c
the following cities in South-East Asia as one
proceeds from South to North? (2014)
1. Bangkok
2. Hanoi
3. Jakarta
4. Singapore
Select the correct answer using the code given
below:
(a) 4-2-1-3
(b) 3-2-4-1
(c) 3-4-1-2
(d) 4-3-2-1

JOIN ME AT TELEGRAM – UPSC PRELIMS WITH NEELESH (AIR 442 UPSC CSE 2021)
https://t.me/UPSCPrelimsWithNeelesh
VISIT OUR WEBSITE – neeleshair442.com ©Copyright reserved with author
6 Which one of the following countries of South- b Jordan, country of South-West Asia does not open out to the
West Asia does not open out to the Mediterranean Sea.
Mediterranean Sea? (2015)
(a) Syria
(b) Jordan
(c) Lebanon
(d) Israel

7 Mediterranean Sea is a border of which of the c


following countries? (2017)
1. Jordan
2. Iraq
3. Lebanon
4. Syria
Select the correct answer using the code given
below:
(a) 1, 2 and 3 only
(b) 2 and 3 only
(c) 3 and 4 only
(d) 1, 3 and 4 only

JOIN ME AT TELEGRAM – UPSC PRELIMS WITH NEELESH (AIR 442 UPSC CSE 2021)
https://t.me/UPSCPrelimsWithNeelesh
VISIT OUR WEBSITE – neeleshair442.com ©Copyright reserved with author
8 Consider the following pairs: (2019) b Bordering nations of:
Sea Bordering country - Adriatic Sea: Albania, Bosnia and Herzegovina, Croatia, Italy,
1. Adriatic Sea : Albania Montenegro, and Slovenia.
2. Black Sea : Croatia
3. Caspian Sea : Kazakhstan - Black Sea: Ukraine to the north, Russia to the northeast, Georgia to the
4. Mediterranean Sea : Morocco east, Turkey to the south, and Bulgaria and Romania to the west.
5. Red sea : Syria
Which of the pairs given above are correctly
matched? - Caspian Sea: Kazakhstan, Turkmenistan, Iran, Azerbaijan, and Russia.
(a) 1, 2 and 4 only - Mediterranean Sea: Albania, Algeria, Bosnia and Herzegovina, Croatia,
(b) 1, 3 and 4 only Cyprus, Egypt, France, Greece, Israel, Italy, Lebanon, Libya, Malta, Monaco,
(c) 2 and 5 only Montenegro, Morocco, Slovenia, Spain, Syria, Tunisia, and Turkey.
(d) 1, 2, 3, 4 and 5
- Red Sea: Egypt, Saudi Arabia, Yemen, Sudan, Eritrea and Djibouti.

9 Consider the following pairs: (2020) c Mekong river flows into the South China Sea.
River Flows into Thames river flows into the North Sea via the Thames Estuary.
1. Mekong : Andaman sea Volga river discharges into the Caspian Sea.
2. Thames : Irish Sea Zambezi river discharges into the India Ocean at the Mozambican coast.
3. Volga : Caspian sea
4. Zambezi : Indian Ocean
Which of the pairs given above is/are correctly
matched?
(a) 1 and 2 only
(b) 3 only
(c) 3 and 4 only
(d) 1, 2 and 4 only

10 Which one of the following lakes of West Africa b Lake Faguibine became entirely dry in the year 2021.
has become dry and turned into a desert? (2022)

JOIN ME AT TELEGRAM – UPSC PRELIMS WITH NEELESH (AIR 442 UPSC CSE 2021)
https://t.me/UPSCPrelimsWithNeelesh
VISIT OUR WEBSITE – neeleshair442.com ©Copyright reserved with author
(a) Lake Victoria Lake Faguibine was a lake in Mali on the southern edge of the Sahara Desert
(b) Lake Faguibine situated 80 km west of Timbuktu and 75 km north of the Niger River to
(c) Lake Oguta which it is connected by a system of smaller lakes and channels. In years
(d) Lake Volta when the height of the annual flood of the river is sufficient, water flows
from the river into the lake. Since the Sahel drought of the 1970s and 1980s
the lake has been mostly dry. Water has only rarely reached the lake and
even when it has done so, the lake has been only partially filled with water.
This has caused a partial collapse of the local ecosystem.

11 Which of the following has/ have shrunk d The Aral Sea was once the fourth largest lake in the world and it produced
immensely/ dried up in the recent past due to thousands of tons of fish for the local economy annually. Since the 1960s,
human activities? (2018) however the Aral Sea has been sinking after the rivers that fed it were
1. Aral Sea diverted by Soviet Irrigation Projects. By 1997, it had declined to 10% of its
2. Black Sea original size, splitting into four lakes. The shrinking of the Aral Sea has been
3. Lake Baikal called “one of the planet’s worst environmental disasters.”
Select the correct answer using the code given
below: Baikal lake is situated in the Russia’s Siberia region. In over the recent few
(a) 1 only years, the water level in the lake has come down drastically.
(b) 2 and 3 The black sea is a body of water and marginal sea of the Atlantic Ocean
(c) 2 only between Eastern Europe, the Caucasus and Western Asia. The Black Sea has
(d) 1 and 3 a positive water balance. Even though, it is expected that due to climate
change, the black sea is losing its habitable waters but it is not shrinking
immensely over the years.

12 Consider the following pairs: (2018) c Catalonia is an autonomous region in Spain on the north-eastern part of the
Region sometimes in News Country Iberian Peninsula.
1. Catalonia : Spain The Crimean Peninsula is connected to mainland Ukraine by two narrow
2. Crimea : Hungary necks of land, making it more like an island with a couple of natural land
3. Mindanao : Philippines bridges than simply a bit of land jutting out into the sea.
4. Oromia : Nigeria Mindanao is the second largest island of the Philippines.
Oromia is one of the nine ethnically based regional states of Ethiopia.

JOIN ME AT TELEGRAM – UPSC PRELIMS WITH NEELESH (AIR 442 UPSC CSE 2021)
https://t.me/UPSCPrelimsWithNeelesh
VISIT OUR WEBSITE – neeleshair442.com ©Copyright reserved with author
Which of the pairs given above is/are correctly
matched?
(a) 1, 2 and 3
(b) 3 and 4 only
(c) 1 and 3 only
(d) 2 and 4 only

13 Consider the following pairs: (2018) b - Aleppo is in Syria.


Towns in News Country - Kirkuk is in Iraq, serving as the capital of the Kirkuk governorate,
1. Aleppo : Syria located to north of Iraq.
2. Kirkuk : Yemen - Mosul is a city located in Iraq, to the north of Baghdad.
3. Mosul : Palestine - Mazar-i-sharif often just called Mazar, is the 4th largest city of
4. Mazar-i-sharif : Afghanistan Afghanistan, and the capital of the Balkh province.
Which of the pairs given above are correctly
matched?
(a) 1 and 2
(b) 1 and 4
(c) 2 and 3
(d) 3 and 4

14 Consider the following pairs: (2016) c The Kurds/ also the Kurdish people are the ethnic group in the middle east,
mostly inhabiting a contiguous area spanning adjacent areas of Eastern and
Southeastern Turkey (Northern Kurdistan), western Iran (Eastern Kurdistan),
Community In the affairs of northern Iraq (Southern Kurdistan), and northern Syria (Western Kurdistan
sometimes in the or Rojava).
news Madhesi are the indigenous ethnic group of Nepalese people who are
Kurd Bangladesh natives of the Madhesh plains of Southern Nepal in Terai belt of South Asia.
Madheshi Nepal Rohingya are Muslim people from the Rakhine State, Myanmar. According
Rohingya Myanmar to Rohingyas and some scholars, they are indigenous to Rakhine State, while

JOIN ME AT TELEGRAM – UPSC PRELIMS WITH NEELESH (AIR 442 UPSC CSE 2021)
https://t.me/UPSCPrelimsWithNeelesh
VISIT OUR WEBSITE – neeleshair442.com ©Copyright reserved with author
Which of the pairs given above is/are correctly other historians claim that they migrated to Myanmar from Bengal primarily
matched? during the period of British rule in Burma.
(a) 1 and 2
(a) 2 only
(b) 2 and 3 only
(c) 3 only

15 Recently, which of the following States has d It was a current affairs based question. Rajasthan government wanted to
explored the possibility of constructing an develop an artificial inland port in Jalore by bringing in Arabian Sea water
artificial inland port to be connected to sea by a into Rajasthan through Gujarat
long navigational channel? (2016)
a. Andhra Pradesh
b. Chhattisgarh
c. Karnataka
d. Rajasthan

16 What is the importance of developing Chabahar c Chabahar Port is a seaport in Chabahar located in southeastern Iran, on the
Port by India? (2017) Gulf of Oman. It serves as Iran’s only oceanic port, and consists of two
a. India’s trade with African countries will separate ports named Shahid Kalantari and Shahid Beheshti, each of which
enormously increase. have five berths.
b. India’s relations with oil-producing Arab In May 2016, India and Iran signed a bilateral agreement in which India
countries will be strengthened. would refurbish any of the berths at Shahid Beheshti port, and reconstruct
c. India will not depend on Pakistan for access to a 600-meter-long container handing facility at the port.
Afghanistan and Central Asia. The port is intended to provide an alternative for trade between India and
Pakistan will facilitate and protect the installation Afghanistan.
of a gas pipeline between Iraq and India Chabahar allows India to side-step Pakistan, which blocks its access to
Afghanistan and Central Asia.

JOIN ME AT TELEGRAM – UPSC PRELIMS WITH NEELESH (AIR 442 UPSC CSE 2021)
https://t.me/UPSCPrelimsWithNeelesh
VISIT OUR WEBSITE – neeleshair442.com ©Copyright reserved with author
Our other initiatives
1. FREE ETHICS AND ESSAY WRITING PRACTICE (TELEGRAM
CHANNEL – ER NEELESH AIR 442 UPSC_CSE2021)
2. FREE SOCIOLOGY ANSWER FRAMEWORK (TELEGRAM CHANNEL
- SOCIOLOGY WITH NEELESH)
3. FREE CSAT MARATHON – TELEGRAM CHANNEL - CSAT WITH
NEEELESH
UPCOMING PLANNARS
INTEGRATED MARATHON FOR 2024 AND 2025 (ON TELEGRAM
CHANNEL – UPSC PRELIMS WITH NEELESH)
SOCIOLOGY LECTURES AND MUCH MORE

JOIN ME AT TELEGRAM – UPSC PRELIMS WITH NEELESH (AIR 442 UPSC CSE 2021)
https://t.me/UPSCPrelimsWithNeelesh
VISIT OUR WEBSITE – neeleshair442.com ©Copyright reserved with author
CIVIL SERVICES WITH NEELESH
(TELEGRAM CHANNEL – UPSC PRELIMS WITH NEELESH)
(YOUTUBE CHANNEL – CIVIL SERVICES WITH NEELESH)
Note – All our initiatives (both free and paid) are on these channels only.

OUR INITIATIVES (Running Currently)

1. PRELIMS MENTORSHIP 2024 (BOTH FREE AND PAID)


2. ETHICS AND ESSAY EVALUATION – ER NEELESH AIR 442 UPSC_CSE2021 (https://t.me/UpscWithNeelesh_AIR442)
3. SOCIOLOGY FRAMEWORK - TELEGRAM – SOCIOLOGY WITH NEELESH (https://t.me/SociologyWithNeelesh)
4. FREE CSAT PROGRAM – ON YOUTUBE CHANNEL – CIVIL SERVICES WITH NEELESH
5. INTEGRATED MENTORSHIP FOR 2025 (PRELIMS AND MAINS) – YEARLONG MENTORSHIP
6. SHORT NOTES FOR ALL SUBJECTS (HANDWRITTEN)

UPCOMING INITIATIVES

1.PRELIMS TEST SERIES FOR 2025


2. MAINS TEST SERIES FOR 2025
3. ESSAY VIDEO BATCH FOR 2024 AND 2025
4. ETHICS VIDEO BATCH FOR 2024 AND 2025
5. SOCIOLOGY VIDEO BATCH FOR 2025 BASED ON PYQ
6. CSAT VIDEO BATCH
7. MAINS MARATHON 2024 AND MUCH MORE
1. Don’t watch the clock; do what it does. Keep going
2. Stay dedicated. It won't happen overnight.
3. Be stronger than your excuses
4. Don’t tell people about your dreams! Show them!!!

JOIN ME AT TELEGRAM – UPSC PRELIMS WITH NEELESH (AIR 442 UPSC CSE 2021)
https://t.me/UPSCPrelimsWithNeelesh
©Copyright Reserved with the Author

You might also like